Zuku Flashcards

1
Q

The 12-year-old spayed female cat is presented with a history of weakness for the past 2 days.

She has neck ventroflexion and a stiff, stilted gait.

Which one of the following is in the top of the differential list?
A - Hypokalemic myopathy
B - Hyperkalemic periodic paralysis
C - Cervical vertebral malformation
D - Caudal occipital malformation
E - Bilateral otitis media/interna
A

a)
Hypokalemic myopathic
The posture is classic for hypokalemia and other neuromyopathies (e.g., myasthenia gravis, organophosphate) in cats.
Hyperkalemis periodic paralysis has not been reported in cats.

How well did you know this?
1
Not at all
2
3
4
5
Perfectly
2
Q

A four-year-old stallion presents with urine scald, fecal retention, tailhead rubbing, and a right head tilt.

Rectal examination reveals atonia of the anus and rectum, however, no fracture is palpated.

Which one of the following tests help provide evidence of the top differential diagnosis?

A - Antibodies against P2-myelin protein
B - Immunoblot against DNA fragments from P. tenuis
C - Search the pasture for yellow star thistle
D - Cervicothoracic spinal radiography
E - Tibial muscle and nerve biopsies

A

A)
Antibodies against P2-myelin protein
The top differential for this horse with cauda equina signs plus cranial nerve signs is polyneuritis equi. Horses with this condition can have circulating antibodies against P2-myelin protein, but many false positives. Other diseases to rule out of this case would be equine herpesvirus-1 and equine protozoal encephalomyelitis.
Unfortunately, there is no definitive Rx and prognosis is poor for functional recovery

How well did you know this?
1
Not at all
2
3
4
5
Perfectly
3
Q

A 15-year-old cat is presented with a three week progression of wandering in circles to the left, standing in corners, and dull mentation.

Conscious proprioceptive deficits (knuckling) can be elicited in the right thoracic and right pelvic limbs.

There is a decreased menace response on the right, but palpebral and pupillary light reflexes are normal in both eyes.

The rest of the examination is normal. A brain tumor is suspected.

Where is the most likely location of the lesion?

A - Right brainstem
B - Right cerebrum
C - Left brainstem
D - Left cerebrum
E - Cerebellum
A

D
Left cerebrum
Remember that conscious proprioception and visual perception are affected by lesions in the contralateral cerebral hemisphere.
Circling direction is generally toward the side of the lesion.
Neurologic lesion localisation can be challenging. Here are some of the summary points that may help you with this case.
What signs make this lesion cerebral?
-Standing in corners (head pressing)
-Wandering in circles to the left (towards side of lesion)
-Contralateral visual loss with normal palpebral and pupillary light reflexes

Why is the lesion not in the brainstem?
-Visual pathways do not course through the brainstem, so the visual loss does not fit

Pearl of wisdom: You could lose the menace response with an ipsilateral facial nerve lesion in the brainstem, but you would not have the wandering and the head pressing

How well did you know this?
1
Not at all
2
3
4
5
Perfectly
4
Q

The menace response is used to test which cranial nerves?

A - II and VII
B - I and VIII
C - V and VI
D - IX and X
E - III and IV
A

A
II and VII
The menace response is used to test visual input (cranial nerve II optic) and blink response (cranial nerve VII facial). Take care not to induce air movement which would elicit a tactile response (cranial nerve V trigeminal) instead of visual.

How well did you know this?
1
Not at all
2
3
4
5
Perfectly
5
Q

A chicken farm has had several ill and dead birds over the past week.

Examination of the sick chickens shows severe depression, ecchymoses and edema of the comb and wattles, green diarrhea.

Some are unable to walk with absent perching reflexes.

Which one of the following choices is the most likely diagnosis?

A - Avian encephalomyelitis
B - Avian influenza
C - Infectious coryza
D - Infectious bronchitis
E - Mycoplasma gallisepticum
A

B
Avian influenza, also called fowl plague, is the most likely diagnosis.
This virus can either be of low pathogenicity (most common) or highly pathogenic.
Viruses of low pathogenicity typically cause subclinical or mild respiratory disease. The case above is due to a highly pathogenic strain of influenza. Infectious bronchitis is characterized by respiratory signs and is a differential for infection with an avian influenza strain of low pathogenicity.

How well did you know this?
1
Not at all
2
3
4
5
Perfectly
6
Q

Which one of the following diseases have been linked with the variant form of Creutzfeldt-Jakob disease (vCJD) in humans?

A - Bovine spongiform encephalopathy
B - Q fever
C - Sporadic bovine encephalomyelitis
D - Equine West Nile virus encephalitis
E - Eastern equine encephalomyelitis (EEE)
A

a)
Bovine spongiform encephalopathy (BSE) suddenly appeared in the 1980s in the UK, likely due to changes in the practice of feeding meat and bone meal of sheep origin. In 1997, it was confirmed that Creutzfedlt-Jakod disease was associated with the bovine disease. Sporadic bovine encephalopathy (SBE), caused by Chlamedia pectoris (also called Chlamedia pecorum), is not known to cause disease in humans

How well did you know this?
1
Not at all
2
3
4
5
Perfectly
7
Q

A pig farmer complains of strange behavior in his feeder pigs.
Most are hyperexcitable yet are not squealing.
A few are lethargic, wandering aimlessly and seem to be blind.
Bloodwork shows a marked increase in sodium concentration.
Upon inspection of the pen, it turns out that the waterer had been inadvertently turned off.
Which one of the following treatments is indicated?

A - Frequent small amounts of water
B - Ad lib water
C - Mannitol
D - Furosemide
E - Ad lib water and IV 1/2 strength Ringers solution
A

a)
Frequent small amounts of water.
It is dangerous to rapidly correct for a hyperosmolar or hypoosmolar state (salt toxicity) and could lead to brain swelling. It is best to gradually rehydrate the animals with small amounts of water given frequently. Also, IV 1/2 strength Ringers solution with 2-1/2% dextrose can be given at a slow rate.

How well did you know this?
1
Not at all
2
3
4
5
Perfectly
8
Q

An intact male whippet presents with an acute onset of swelling and pain in one of his testicles.

Which one of the following tests is helpful to diagnose the cause of the swelling before surgery?

A - Ultrasonography
B - Lateral radiography
C - Magnetic resonance imaging
D - Nuclear scintigraphy scan
E - Positive contrast cystography
A

a)
A skilled ultrasonography can differentiate between testicular torsion, scrotal hernia, or focal lesions in the testicle.

How well did you know this?
1
Not at all
2
3
4
5
Perfectly
9
Q

A horse recently imported from Dubai died after a being ill for about 2 weeks in a quarantine facility. He was febrile and very depressed, with dyspnea and respiratory distress in the last 24 hours.

Other signs included hyperemia of the conjunctiva and edema all over the head and neck, especially the eyelids and the supraorbital fossae. Necropsy reveals a clear pericardial effusion, froth in the upper airways, and heavy distended lungs.

Which one of the following choices is the most likely diagnosis?

A - Equine influenza
B - Equine infectious anemia
C - African horse sickness
D - Equine viral arteritis
E - Piroplasmosis
A

c)
African horse sickness (AHS) presents with fever, respiratory signs, ans deem. Edema is a prominant clinical sign, especially of the head and neck. Those with respiratory signs die within about a week; death occurs after 1-2 weeks in the cardiac form of the disease. Caused by an orbivirus, AHS is transmitted via bites of infected Culicoides app., most often during warm, humid seasons. AHS is very lethal, especially in naive populations, where mortality approaches 90%. Pulmonary deem and hydropericardium are a commun necropsy findings. No specific treatment is effective, but attenuated-live virus vaccines are available that are protective. The disease is endemic in southern Egypt and occurs sporadically in many parts of Africa. AHS has never occurred in the United States and is a reportable disease. Other differentials include anaplasmosis, piroplasmosis, equine viral arteritis ans Equine infectious anemia.

How well did you know this?
1
Not at all
2
3
4
5
Perfectly
10
Q

Which of the following is the correct location for placement of a proximal paravertebral block used to perform a standing laparotomy in a cow?

A - In the spaces between L1, L2, L3, and L4
B - Above and below the transverse processes of L1, L2 and L4
C - Midway between spine and the ends of the transverse processes of L1, L2, L3
D - Caudal to the transverse processes of T13, L1, and L2
E - At the ends of the transverse processes of L1, L2, and L4

A

d)
A proximal paravertebral block is performed via placement of local anesthetic just off the midline and caudal to the transverse processes of T13, L1, and L2 vertebrae. With a distal paravertebral block, local is placed above and below the ends of the ends of the transverse processes of L1, L2, and L4 vertebrae. The spinal nerves, T13, L1, and L2 are targeted by a paravertebral block to completely desensitize the flank of a cow. Note the difference- proximal at T13, L1, and L2, distal at L1, L2, and L4, but the same nerves are affected because they gradually course caudally after exiting their spinal foramen. Proper placement results in warming of the skin from vasodilation, anesthesia of the skin and body wall, and a curvature of the spine in some cows. The latter is caused by relaxation of the epaxial musculature on the affected side; the spin curves towards the opposie side.

How well did you know this?
1
Not at all
2
3
4
5
Perfectly
11
Q

During the necropsy of an 8-year-old mixed breed dog from the Southern United States, reactive granulomas in the esophagus containing bright red worms, 40 mm to 70 mm long are found.

Which choice is the most likely diagnosis?

A - Ollulanus tricuspis 
B - Haemonchus placei 
C - Spirocerca lupi 
D - Gastrophilus spp.
E - Physaloptera spp.
A

c)
A disease of dogs in the Southern U.S and tropical climate, Spirorcerca lupine (esophageal worms) make reactive granulomas of variable size in the esophageal, gastric or aortic walls. Spirocercosis may also lead to aneurysm in the thoracic aorta or an ossifying spondylitis of the posterior thoracic vertebrae. Typically asymptomatic, but large granulomas can cause esophageal obstruction. Large granulomas may become neoplastic (osteosarcoma, fibrosarcoma). Some dogs develop spondylitis or enlargement of the extremities characteristic of hypertrophic osteopathy.
All of the other choices are gastric parasites. Ollulanus tricuspid is an uncommon gastric parasite of cats. Physaloptera app. is a stomach worm the may cause vomiting, anorexia, dark fèces in dogs and cats. Haemonchus ss. ostertagia spp. and Trichostrongylus spp. are found in the abomasum of ruminants. Gastrophilus spp. are the larvae of horse bot flies, found in the stomach of horses.

How well did you know this?
1
Not at all
2
3
4
5
Perfectly
12
Q

Many hens in a large backyard flock recently became ill after the owner purchased four new chickens. The sick hens are gasping, coughing and sneezing.

They have poor appetite and act depressed. Some affected birds have watery green diarrhea and swelling of the head and neck.

A few have paralyzed legs and wings, twisted necks, are circling, and have tremors or clonic spasms.

Birds are not laying well and some eggs are misshapen with watery albumen. Many of the sickest birds have died.

Necropsy of a dead chicken shows petechial hemorrhages on the mucosal surface of the proventriculus and gizzard.

What should be done next to address this problem?

A - Increase poultry house temperatures to reduce morbidity
B - Disinfect housing with phenolic compounds, barrier precautions for staff
C - Treat all affected chickens with amprolium
D - Cull affected birds and vaccinate the remainder
E - Report outbreak to regulatory authorities

A

e)
This history and the necropsy findings are consistent with viscerotropic velogenic Newcastle disease (VVND), which is reportable. The appropriate regulatory authorities should be contacted ASAP. Gross lesions such as the petechiae seen here in the pro ventricular mucosa are not usually seen with low-virulence Newcastle disease (loNDV). the clinical history is also suggestive of VVND. The acute from a of NDV must be differentiated from highly pathogenic avian influenza by isolation of a hem agglutinating virus identified by inhibition with Newcastle disease antiserum. NDV occurs worldwide and chickens usually present with acute respiratory disease. Occasionally diarrhea, neurological problems, or depression predominate. Virulence varies between the VVND and loNDV forms. Vaccines against NDV decrease clinical signs and death. Infected poultry are depopulated. Sometimes the temperature of the poultry house is increased to decrease morbidity in flocks with suspected avian influenza. Amprolium is a treatment for coccidiosis.

How well did you know this?
1
Not at all
2
3
4
5
Perfectly
13
Q

Many hens in a large backyard flock recently became ill after the owner purchased four new chickens. The sick hens are gasping, coughing and sneezing.

They have poor appetite and act depressed. Some affected birds have watery green diarrhea and swelling of the head and neck.

A few have paralyzed legs and wings, twisted necks, are circling, and have tremors or clonic spasms.

Birds are not laying well and some eggs are misshapen with watery albumen. Many of the sickest birds have died.

Necropsy of a dead chicken shows petechial hemorrhages on the mucosal surface of the proventriculus and gizzard.

What should be done next to address this problem?

A - Increase poultry house temperatures to reduce morbidity
B - Disinfect housing with phenolic compounds, barrier precautions for staff
C - Treat all affected chickens with amprolium
D - Cull affected birds and vaccinate the remainder
E - Report outbreak to regulatory authorities

A

e)
This history and the necropsy findings are consistent with viscerotropic velogenic Newcastle disease (VVND), which is reportable. The appropriate regulatory authorities should be contacted ASAP. Gross lesions such as the petechiae seen here in the pro ventricular mucosa are not usually seen with low-virulence Newcastle disease (loNDV). the clinical history is also suggestive of VVND. The acute from a of NDV must be differentiated from highly pathogenic avian influenza by isolation of a hem agglutinating virus identified by inhibition with Newcastle disease antiserum.

How well did you know this?
1
Not at all
2
3
4
5
Perfectly
14
Q

A two-year-old heifer is presented who died the night before.

She was stunted compared to her herdmates, and had a three-week history of poor appetite, unthriftiness, diarrhea.

Necropsy reveals that the abomasum is edematous and covered in small umbilicated nodules 1-2 mm in diameter (cobblestone or “Moroccan leather” appearance).

Several other younger cows in this Spring-calving herd look unthrifty and have diarrhea too.

Which one of the following options is the most consistently effective against all stages of this parasite?

A - Amprolium
B - Thiabendazole
C - Levamisole
D - Ivermectin
E - Morantel
A

d)
Avermectins (macrocyclic lactones), such as ivermectin, are the most consistently effective of theses choices against all stages, including hypo biotic larvae, of Ostertagia, the parasite in this calf’s gastrointestinal tract. An abomasum with a cobblestone or “Moroccan leather” appearance is pathognomonic for Ostertagia, one of the three stomach worms of cattle. In order to minimize the development of anthelmintic resistance, current recommendations for parasite control include not only appropriate deworming, but adequate nutrition and rotation of treated stock to “clean” pastures. Some experts recommend deworming of only the most severely affected animal in a group (targeted treatment). The idea is to slow development of drug-resistant populations by allowing some worms to survive (these are called “refugia”). If some worms survive that are susceptible to antiparasitiez drugs, it decreases population pressure to develop drug resistance.
Amprolium is your treatment of choices for calves and cows with coccidiosis.

How well did you know this?
1
Not at all
2
3
4
5
Perfectly
15
Q

Which one of the following choices includes the cardinal sign of trigeminal neuritis?

A - Masseter muscle pain associated with chewing
B - Circling and head tilt toward side of lesion, no other signs
C - Dysphagia, dysphonia and stridor
D - Inability to close the mouth
E - Paralyzed eyelid, ear or lip on one or both sides of the face

A

d)
Idiopathic trigeminal neuritis, due to inflammation of cranial nerve 5 (CN 5), is characterized by acute onset od flaccid jaw paralysis. Affected animals cannot close their mouth and have difficulty eating and drinking. Seen occasionally in dogs, rare in cats. Cause is unknown.
Idiopathic facial nerve paralysis, affecting cranial nerve 7 (CN7), results in the inability to move the eyelid, lip, or ear and dryness of the eyes and mouth.
Masticatory myositis is characterized by pain on opening the mouth and swelling of the uses of mastication (acute) or atrophy of the temporals and master muscles with the inability to open the mouth due to fibrosis (chronic).
Dysphagia, dysphoria, and stridor are most often associated with dysfunction of the vagus nerve, cranial nerve 10 (CN 10).
Circling and head tilt toward the side of the lesion with no other signs is a common presentation of vestibulocochlear nerve, cranial nerve 8 (CN 8) lesions.
Concurrent CN 7 paralysis and Honrer’s syndrome (ptosis, miosis, enophthalmis) may be present with middle- and inner-eat infections.

How well did you know this?
1
Not at all
2
3
4
5
Perfectly
16
Q

What kind of organism causes equine granulocytic ehrlichiosis?

A - Spirochete
B - Anaplasma 
C - Ehrlichia 
D - Protozoa
E - Chlamydia
A

b)
This is a tricky question to help you remember that two diseases formally classified as Ehrlichia have now been re-classified. The causative organism of equine granulocytic ehrlichiosis (EGE) was originally classified as Ehrlichia equivalent, but i now called Anaplasma phagocytophilum due to DNA sequencings studies. Don not confuses EGE, a necrotizing vasculitis in horse with bovine anaplasmosis, primarily an anemia, with icterus, fever.Another name change occurred with the causative organism of Potomas horse fever (PHF), from Ehrlichia risticii to Neorickettsia risticii. PHF presents as a febrile colitis/diarrhea, with laminitis 3-5 d after diarrhea in horses of all ages: A big rule out is salmonella (think septicaemia/fever+ diarrhea).

How well did you know this?
1
Not at all
2
3
4
5
Perfectly
17
Q

Failure to control mild endemic respiratory disease in swine caused by Mycoplasma hyopneumoniae predisposes pigs to complications.

Exudative bronchopneumonia and polyarthritis are most commonly seen in herds infected with both Mycoplasma hyopneumoniae AND which other organism?

A - Bordetella bronchiseptica 
B - Fusobacterium necrophorum 
C - Hemophilus parasuis 
D - Pasteurella multocida 
E - Swine influenza virus
A

d)
Pasteurella multocida infection in conjunction with Mycoplasma hyopneumoniae causes exudative bronchopneumonia, polyarthrites, and chronic lung lesions. Mycoplasma hyopneumoniae (also called “enzootic pneumonia”) is a commons, smouldering low-level illness. Stressors (parasites, other infections, even weather) can result in severe pneumonia. Best managed by decreasing stressors with improves ventilation and decreased overcrowding. In endemic herds, ABX for sick individuals (i.e.: lincomycin, tyrosine, tiamulin or a tetracycline) helps control illness, mostly by preventing secondary bacterial infection (like Pasteurella). Bacterin vaccines give good protection, decrease signs (coughing). Pre-farrowing vaccination of sows decreases colonization in sucklings piglets.

How well did you know this?
1
Not at all
2
3
4
5
Perfectly
18
Q

At what age do a cow’s permanent canine teeth begin to erupt?

A - Cows don't have canines
B - 2 to 2.5 years
C - 3 years
D - 3.5 to 4 years
E - 5-6 years
A

D)
3.5 to 4 years. Cows do have canine teeth but the look like incisors. References vary about six months, because the times of tooth eruption also vary among different indival animals. 1st permanent incisors (I1-centrals) erupt at 1 to 1.5 years. 2nd permanent incisors (I2-medials) erupt ar around 2 to 2.5 years. I3-laterals erupt at roughly 3.5 years. Permanent canines (which look like a 4th set of incisors in ruminants) come in around 3.5 to 4 years. Full eruption of a new tooth may take another six months. One reference, Pasquini, suggests this approximate, but easy-t-remember rule of thumb for fully erupted permanent incisors in cows:I1-2yr, 12-3 years I3-4yr.

How well did you know this?
1
Not at all
2
3
4
5
Perfectly
19
Q

Imidocarb diproprionate is the most effective therapeutic for which one of the following organisms?

A - Bartonella spp.
B - Babesia canis
C - Rickettsia rickettsii
D - Borrelia burgdorferi
E - Trypanosoma congolense
A

b)
Imidocarb diproprionate is the treatment of choice for Babesia canis but is not effective in clearing B. gibsoni and B. felis. B. gibsoni is usually susceptible to treatment with atovaquone and azithromycin.
Two species of Babesia, B. gibsoni (small babesia) and B. canis (large babesia), are important causes of babesiosis in dogs. Babesiosis is a tick borne disease -clinicla signs of hemolytic anemia and thrombocytopenia. Polymerase chain reaction is the only conclusive way to determine which Babesia species is responsible.

How well did you know this?
1
Not at all
2
3
4
5
Perfectly
20
Q

Two calves aged eight weeks are presented that are both down and extremely weak.

They are depressed and lying in pools of foul-smelling brown diarrhea with a small amount of blood.

One other calf died suddenly the night before with no signs at all.

They are found to be dehydrated, with rectal temperatures of 105.2 and 105.6 F (40.7 – 40.9 C)..[N=101.5-103.5F, N=37.8-39.7 C], respectively.

Based on the condition at the top of the differential diagnosis list, what is the treatment plan?

A - Immunize well and sick calves and adult cattle with MLV vaccine, antibiotics for sick calves
B - NSAIDS, antibiotics in feed and water
C - IV fluids, NSAIDS, fequent milk feeding, antibiotics if septic
D - Cull sick calves, disinfect feeding areas, prophylactic antibiotics and NSAIDs for well animals
E - Isolate sick calves, immunize, antibiotics, cull those that do not respond to treatment

A

C)
IV fluids, NSAIDS, frequent milf feedings and antibiotics.
Fever, diarrhea and sudden death in 8-week-old calves is highly suggestive of septicaemia due to salmonellosis. Isolate sick calves. Use of antibiotics is controversial as they may prolong recovery and shedding and yield a carrier calf. However if an animal is septic it needs parenteral antibiotics. According to Pasquini’s, prognosis is poor with neonatal salmonella and deaths can approach 100% in affected calves. In adults, antibiotics may yield a clinical cure, but Salmonella can establish in biliary system andintermittently shed into GI system, leading to environnemental contamination. Prevention is dependant on which species of Salmonella is causing the problems-host adapted or environmental.

How well did you know this?
1
Not at all
2
3
4
5
Perfectly
21
Q

Which one of the following hormones can induce neoplastic transformation of hypertrophied mammary tissue in cats?

A - Oxytocin
B - Progesterone
C - Estrogen
D - Testosterone
E - Prolactin
A

b)
Exogenous progesterone administration can lead to neoplastic transformation of mammary hyperplasia in intact or neutered male or female cats. Patients undergoing progesterone therapy that develop mammary hyperplasia should have enlarged mammary gland(s) removed and submitting for histology in addition to the cessation of the progesterone therapy.

How well did you know this?
1
Not at all
2
3
4
5
Perfectly
22
Q

Which organism determines the minimum temperature needed to pasteurize milk in the United States?

A - Mycobacterium bovis 
B - Listeria monocytogenes 
C - Brucella abortus 
D - Escherichia coli 
E - Coxiella burnetii
A

E)
Temperature of pasteurization is set by the toughest, most heat-resistant organism, which is Coxiella burnett, the cause of Q fever. If the temperature is high enough, for long enough to kill Coxiella, it will also kill the other organisms. Basically, the higher the temperature, the shorter the pasteurization time needed. i.e..: 90 C(194 f) for 0.5 seconds, or 100 C(212 F) for 0.01 seconds.
Brucella and Listeria can also be transmitted in milk. E. coli is typically a contaminant forma unsanitary milking conditions.

How well did you know this?
1
Not at all
2
3
4
5
Perfectly
23
Q

Acromegaly is associated with what 3 diseases in cats?

A - Hypoadrenocorticism, Congestive heart failure, Hepatic lipidosis
B - Hyperadrenocorticism, Pleural effusion, Lameness
C - Diabetes mellitus, Cardiomyopathy, Renal disease
D - Prognathism, Pulmonary edema, Pulmonary Hypertension
E - Vertebral spondylosis, 3rd degree heart block, Glomerulonephritis

A

c) Diabetes mellitus, cardiomyopathy and Renal disease
Think first of Diabetes mellitus in older male cats (ave. 10 yr, 90% male) with feline acromegaly. First presenting sign may be PU/PD, polyphagia of diabetes. Weigth gain in an unregulated diabetic cat strongly suggests acromegaly. Think also of Cardiomyopathy (50%, see weakness, dyspnea, systolic murmur, cardiomegaly/CHF, pulmonary edema). Think also of renal disease (50%, see proteinuria, USpG 1.015-1.025, (ans glycosuria from DM)). May see prognathism (long mandible), lameness (est. cats), marked vertebral spondylosis.

How well did you know this?
1
Not at all
2
3
4
5
Perfectly
24
Q

What percentage of dairy cows have a retained placenta after calving?

A - 1-4%
B - 5-15%
C - 25%
D - 20-30%
E - 30-40%
A

b)
Retained placenta occurs in 5-15% of dairy cattle. Very common, about 1 in 10 callings. Normally, the placenta is expelled in 3-8 hours. If it’s still there 12-24 hours after calving, it’s retained. Typically, retained placentas are dispelled spentaneously after 4-10 days as the caruncles necroses. An owner may insist you pull it out. manual removal is usually not helpful-shout only be done if placenta comes out with GENTLE traction. Manual removal is contraindicated if cow has signs of septicaemia- removal may cause a septic metritis, peritonitis.

How well did you know this?
1
Not at all
2
3
4
5
Perfectly
25
Q

What are ongoing losses in fluid therapy?

A - Urine output
B - Fluids given minus extracellular volume estimate
C - Fluid lost during panting
D - Vomit and diarrhea
E - Percent dehydration multiplied by body weight

A

D)
Ongoing losses are typically vomiting, diarrhea or other losses into a third compartment (i.e. pleural or abdomen cavity, subcutaneous space). These are sometimes called contemporary losses. A classic example of a disease with significant ongoing losses is a puppy with parvovirus who has both diarrhea ad vomiting or a horse with acute colitis. Sensible losses are easily measured, like the fluids the animal loses by urination. Insensible losses are the fluids an animal loses that are hard to measure, through breathing and via the skin. In humans and animals that sweat (like horses), sweating is part of insensible losses. Remember that all three types of loss must be accounted for (or at least estimated) when calculating fluids needed to rehydrate a dehydrated animal.

How well did you know this?
1
Not at all
2
3
4
5
Perfectly
26
Q

Retained placenta and metritis can predispose to which one of the following secondary conditions in horses?

A - Laminitis
B - Postpartum dysgalactia syndrome
C - Contagious equine metritis
D - Cystic endometriosis
E - Colic
A

a)
In horses, retained placenta and septic metritis can cause acute laminitis. Other causes of equine laminitis include: gain overload, overeating lush spring grasses, insulin dysregulation, colic, endotexemia, exposure to black walnut (Juglans migra) shavings, over-exercice on hard surfaces, lameness on one limb requiring overdependency on opposite leg, which then develops laminitis

How well did you know this?
1
Not at all
2
3
4
5
Perfectly
27
Q

A large male guinea pig from a group of four is presented with swollen and scabby hind feet from which Staphylococcus aureus is cultured.

Which one of the following recommendations is the most appropriate choice?

A - Isolate affected animal, treat with oral amoxicillin
B - Improve sanitation, install smooth-floored enclosure
C - Cull affected animal, increase ventilation of environment for remaining animals
D - Tetracycline-medicated water for all animals
E - Debridement and topical 1% butenafine cream

A

b)
The preferred answer is to improve sanitation and install a smooth-floored enclosure. Bumblefoot (pododermatitis) in guinea pig usually occurs secondary to poor sanitation, obesity and wire cage floors or rough bedding. If detected early, switch to smooth-bottom flooring, keep the enclosure clean, and change to a softer bedding. Chlorexidine soaks and debridement can help the feet. Prognosis is guarded. Avoid penicillins in guinea pigs.

How well did you know this?
1
Not at all
2
3
4
5
Perfectly
28
Q

A five-year old lactating Holstein dairy cow is being evaluated for a two-month history of watery diarrhea, weight loss, and decline in milk production.

On examination, the cow is bright, alert and responsive, but unthrifty and very thin, with a soft intermandibular swelling.

What is the most appropriate management of this herd?

A - Cull positives, ensure 4 L. colostrum uptake for calves
B - Quarantine herd, treat until herd tests negative
C - Test herd, treat positive animals, add coccidiostat to ration
D - Test herd, cull positives, remove calves from dam at birth
E - Test herd, treat positive animals, test and treat replacements

A

d)
Test herd, cull positives, remove claves from dam at birth
This is likely to be Mycobacterium avium subspecies paratuberculosis (john’s disease). NO Rx: test-and-cull programs offer best chance of attaining a Mycobacterium avium subspecies paratuberculosis-free herd. Calves, kids, or lambs should be birthed in manure-free areas. Fro dairy cattle, calves should be removed from dam immediately after birth, bottle-fed pasteurized colostrum or colostrum from test-negative dams. They should be segregated as much as possible from adult cows and their manure until more than one year old. Must also remove calves at birth from the dam to prevent. REPORTABLE in many states

How well did you know this?
1
Not at all
2
3
4
5
Perfectly
29
Q

A female show dog is approaching estrus and the owner has consulted you about planning a breeding for this dog.

She is aware of the signs of proestrus and plans to contact you when those are apparent in her dog.

In addition to vaginal cytology, which one of the following will be the most useful to you in planning the timing of breeding?

A - Vaginoscopy
B - Determination of serum estrogen levels
C - Timed administration of luteinizing hormone
D - Serial progesterone testing
E - Ovarian ultrasonography

A

d)
Serial progesterone testing is the most commonly used hormonal assay method for estimation of the approximate ovulation day in bitches. Progesterone levels are typically measured every two or three days once significant cornification is seen in the vaginal cytology of a prostrés bitch.Baseline progesterone is 0 to 1 ng/mL. About 48 hours prior to ovulation, a rise in progesterone (greater than 2 ng/ml) is seen, corresponding with the luteinizing hormone (LH) surge, which is the actual stimulus for ovulation. The progesterone level increases to 4-10ng/ml (or in some reports up to 15 ng/ml) on ovulation day. For natural breeding or insemination with fresh or chilled semen, dogs are usually bred at least twice, typically on days 2 and 4 after the estimated ovulation date. Serial daily measurement of serum LH is also a recognized method of estimation of ovuation day in bitches and is in fact the most definitive diagnostic test available. However, LH testing is hampered by expense, necessity for daily testing, and problems with availability of tests. Administration of exogenous LH has not been shown to be safe and effective in inducing estrus in dogs. While vaginoscopy is a useful adjunct in breeding management, progesterone measurement is a more accurate method. Estrogen measurement is of little value for estimation of ovulation day because peak levels vary from dog to dog and do not necessarily correlate with the fertile period. Ovarian ultrasonography is not commonly used in clinical practice in dogs.

How well did you know this?
1
Not at all
2
3
4
5
Perfectly
30
Q

For the last 10 years, your state has had a mandatory vaccination program against “pedunculated giblet disease” in fur-bearing turtles and the prevalence of this terrible disease has decreased markedly.

How does this decrease in prevalence affect the predictive value positive (PVP) of the best serologic test for pedunculated giblet disease?

A - PVP depends on the number tested, not prevalence
B - PVP is affected by specificity, not prevalence
C - PVP increases as prevalence decreases
D - PVP decreases as prevalence decreases
E - PVP stays the same as prevalence decreases

A

d)
Don’t think too hard on this: As prevalence of a disease goes down, PVP of your serologic test also goes down. That is, as your disease becomes more and more rare, the predictive value of your same old test gets worse. That is all you need to know. Read more only if you want to see the math (but you don’t need it).
Lets say prevalence of a pedunculate giblet disease is 30% in 1000 turtles (that’s 300 infected, 700 disease-free, then). A test with 90% sensitivity would correctly Dx 270 (cell “a”) with the disease (true pos) and correctly say 30 were negative (cell “c”). A test with 90% specificity would correctly would correctly Dx 630 (cell “d”) as disease-free (true negs) and correctly say 70 were positive (cell “b”): PVP=a/(a+b)=210/(270+70)=79%.

How well did you know this?
1
Not at all
2
3
4
5
Perfectly
31
Q

Acquired equine motor neuron disease is a disease characterized by weight loss, paresis with trembling, preference for lying down, and decreased activity.

Which one of the following choices is associated with this disease?

A - Thiamine deficiency
B - Vitamin E deficiency
C - Lead toxicosis
D - Organophosphate toxicosis
E - Rickettsial infection
A

b)
Vitamin E deficiency. Acquired equine motor neurone disease is uncommon today due to horse owner awareness. When it occurs, it is usually seen in older adult horses. Owners may say that the horse can “walk better than it can stand” when a horse has acquired equine motor neurone disease. The horse will stand still trembling in a tucked up stance, but it will walk well (but in short strides). Sometimes the horse has an “elephant on a ball” stance.

How well did you know this?
1
Not at all
2
3
4
5
Perfectly
32
Q

A four-year-old Rat Terrier is presented with a three day history of progressive stumbling and falling.

Physical exam reveals a right head tilt, left sided hypermetria, generalized ataxia and vertical nystagmus.

Which one of the following choices is at the top of the differential list?

A - Central pontine myelinolysis
B - Fibrocartilagenous embolism
C - Ascending and descending myelomalacia
D - Granulomatous meningoencephalitis
E - Amyotrophic lateralizing sclerosis
A

d)
granulomatous meningoencephalitis. The lesion in this case localizes to the left cerebellum causing the left hypermetria and a right paradoxical head tilt. Granulomatous meningoencephalitis often affects this area. Another top differential would be a cerebellar neoplasm.

How well did you know this?
1
Not at all
2
3
4
5
Perfectly
33
Q

In September, a 15-year-old Quarterhorse mare is presented with a five-day history of left head tilt, facial paralysis, depression, and stumbling.

The horse is ataxic and knuckles on both forelegs, worse on the left.

There is muscle atrophy of the left pectorals and right hindquarters and strips of localized spontaneous sweating over the left trunk.

A serum:CSF ratio is consistent with a diagnosis of Sarcocystis neurona.

What prognosis is given to the owner if she elects to treat the horse?

A - Only 25% of treated horses improve in their neurologic state
B - 50% of treated horses recover completely
C - Prognosis is good if treated with doxycycline
D - At least 60% of treated horses improve
E - Prognosis is grave

A

d)
At least 60% of treated horses improve. The prognosis is guarded to fair for treatment of equine protozoal myeloencephalitis (EPM). Less than 25% recover completely, but at least 60% of treated horses improve. affected animals typically improve by 1-2 ataxia grades (on the mayhem ataxia scale of 1-5). The earlier treatment is started, the better the outcome. 10-20% of horses relapse within 2 years of initial diagnosis.

How well did you know this?
1
Not at all
2
3
4
5
Perfectly
34
Q

Which one of the following organisms transmits the causative agent of blackhead disease in turkeys?

A - Heterakis gallinarum 
B - Ascardia galli 
C - Capillaria spp
D - Eimeria spp
E - Raillietina spp
A

a)
The cecal nematode Heterakis gallinarum transmits Histomona meleagridis (a protozoan), the causative agent of blackhead disease. Clinical signs of histomoniasis include listlessness, droopy wings, unkempt feathers, yellow droppings. Clinical signs of Ascardia galli infection include diarrhea, anemia, and decreased egg production. Birds infected with Capillaria or Eimeria have hemorrhagic enteritis. Clinical signs of Raillietina (tapeworm) infection range from none to enteritis.

How well did you know this?
1
Not at all
2
3
4
5
Perfectly
35
Q

In which one of the following cattle breeds is syndactyly most commonly inherited?

A - Brown Swiss
B - Angus
C - Hereford
D - Holstein-Friesian
E - Simmental
A

d)
Holstein-Friesian cattle inherit syndactyly, or “mule foot”, a simple autosomal recessive trait, more often than other breeds. Syndactyly is the partial or complete fusion of the digits of one or more feet. Forefeet are affected most often. Affected animal walk slowly, usually with a high-stepping gait. They may also be more prone to hyperthermia than normal cows.

How well did you know this?
1
Not at all
2
3
4
5
Perfectly
36
Q

Which one of the following choices is a cardinal and early sign of toxicoinfectious botulism in a foal?

A - When startled, affected foals fall and exhibit opisthotonos
B - Milk dribbles from mouth and nose; foal cannot retract tongue
C - Sawhorse stance, trismus, bilaterally prolapsed nictitans (3rd eyelid)
D - Frequent chewing motions, hypersalivation, vocalizing
E - Hemorrhagic enterocolitis with headpressing

A

b)
Dribbling milk from the nose and mouth with a weak, non retractable tongue is a cardinal and early sign of toxicoinfectious botulism (shaker foal syndrome). In foals most often occurs due the bacteria entering via a moist umbilicus. Affected foals demonstrate signs of progressive symmetric motor paralysis, including stilted gait, muscular tremors, and frequent recumbency. Mortality is higher than 90%; may find foal dead or disease can progress over a week. Most often seen in foals 1-2 months of age.

How well did you know this?
1
Not at all
2
3
4
5
Perfectly
37
Q

Which one of the following would be the best way to prevent a vibriosis abortion storm in ewes?

A - Vaccination program
B - Separate lambs from ewes and bottle-feed colostrum-replacer
C - Cull affected ewes
D - Twice yearly ivermectin 
E - Chloramphenicol in the water
A

a)
Vaccination program. Ewes in flocks with vibriosis-associated abortion should be vaccinated annually 2 weeks prior to breeding.If it’s the ewe’s first vaccine, give a booster mid-pregnancy. Vibriosis (Campylobacter fetus fetus, C. jejuni jejuni, and C. lari) is associated with late-term abortion or stillbirths; fetus usually autolyzed. It is orally transmitted. Outbreaks tend to occur ever 4-5 years. dx- dark field or fluorescent antibody of placental tissue: uterine discharge; fetal abomasa contents, liver, or lungs. Long acting oxytetracycline are effective against some C. jejuni and C. fetus fetus, but there us increasing resistance in US sheep operations. Daily dihydrostreptomycin or penicillin can also be used but is more labor intensive. Strict hygiene is critical in stopping an outbreak. Chloranphenicol is prohibited in food animals. C.jejuni is zoonotic.

How well did you know this?
1
Not at all
2
3
4
5
Perfectly
38
Q

A cow is presented on emergency with urea/non-protein nitrogen toxicity.

What is the treatment of choice?

A - Rumenotomy
B - Relieve bloat, drench with 2-8 liters sodium bicarbonate
C - IV Fluids with MgSO4, Na thiosulfate PO
D - Rumenal infusion 2-8 liters vinegar,3-10 gallons cold water
E - Atropine, Protopam chloride IV q 4-6 hours

A

d)
Treat urea/non-protein nitrogen (NPN) toxicity with a luminal infusion 2-8 liters 5% acetic acid (vinegar) and 3-10 gallons of cold water. The vinegar decreases luminal pH which slows absorption of un-ionized ammonia. Repeat Q 6 hours up to 48 hours. Best results if animal is still ambulatory. It is often impossible to treat these cases before they die because of rapid progression to death. If possible, Rx with IV fluids. If necessary, relieve bloat. Urea/NPN toxicity is related to ammoniated feed toxicity which causes so-called “bovine bonkers”. Manage ammoniated feed toxicity by removing the ammoniated feed and treating severe cases with anticonvulsants like diazepam.

How well did you know this?
1
Not at all
2
3
4
5
Perfectly
39
Q

Under what conditions is a very sensitive test used?

A - Rare disease, Early diagnosis improves prognosis
B - Lethal disease, Highly prevalent disease
C - Treatment does not affect prognosis, Non-infectious diseases
D - Common disease, infectious diseases
E - Zoonoses, untreatable diseases

A

a)
You need a very sensitive test if: disease is rare (e.g.:BSE), early Dx improves prognosis (e.g.: HIV in people), the disease is highly lethal or consequences of missing a case are severe (e.g.: rabies, brucellosis, BSE, screw-worm, FMD, EIA).
Remember that a highly sensitive test will have very false negatives. That means if a test is highly sensitive, you can trust a negative test.

How well did you know this?
1
Not at all
2
3
4
5
Perfectly
40
Q

A commercial turkey operation experiences an outbreak of disease in 40% of its 12-16 week old birds. Typically affected birds show only depression, then death.

Some animals appear weak, recumbent with one leg forward and one leg back.

On necropsy, diffuse or nodular lymphoid tumors are visible in various organs, particularly the liver, and spleen. Two affected birds have distorted pupils.

What is the diagnosis?

A - Viral hepatitis of turkeys
B - Newcastle disease
C - Marek's disease
D - Infectious bursal disease
E - Histomoniasis
A

c)
Yes, turkeys can get Marek’s disease. A transient paralysis with one leg forward, one leg back is a classic sign of Marek’s-just don’t expect to see it in all birds. Typically affected birds show only depression, then death. You may see diffuse or nodular lymphoid tutors are visible in various organs, particularly the liver, spleen and other organs. Lymphoid infiltrates can cause distortion of the pupil and enlargement of feather follicles (“skin leukosis”=condemnation of carcasss).

How well did you know this?
1
Not at all
2
3
4
5
Perfectly
41
Q

An unweaned two-month old calf is presented for necropsy.

The calf collapsed and suddenly died after she escaped and was chased around the back pasture for 45 minutes by her owner.

Among other things, white myocardial and endocardial streaking in the left ventricle of this calf’s heart are evident.

What advice should be given the farmer?

A - Put all the calves off ionophore-containing feed
B - Start the other calves on Ceftiofur (Naxcel®)
C - Treat the other calves with vitamin E/Selenium
D - Check the mother for bovine leukosis by AGID
E - Search the calf barn for sources of lead

A

c)
Treat the other calves with vitamin E/selenium.
Think of white muscle disease when you see sudden death and endocardial plaques in a young calf, lamb or kid with a history of recent vigorous exercise. Typically seen in young, fast growing animals (i.e.: calves 2 weeks-6 months). Clinical signs may include dyspnea (due to myocardial disease), stiff gait, arched back, weakness, recumbent but BAR (bright, alert, responsive). Sudden death may resemble enterotoxemia, should see acute bloody diarrhea, convulsions, opisthotonos in first days of life with enterotoxemia.

How well did you know this?
1
Not at all
2
3
4
5
Perfectly
42
Q

Which of the following intravenous solutions are hypotonic?

A - 0.45% Saline, 5% dextrose
B - Lactated ringers solution, Normosol-R®
C - Hetastarch, Vetstarch
D - Plasmalyte 148, Normosol-M®
E - 5% Dextrose in 0.9% saline
A

a)
0.45% and 5% dextrose are hypotonic fluids. The osmolality of half-strength or 0.45% saline is 155, dextrose is 252 milliosmoles/L (and the dextrose is quickly metabolized, leaving free water). The calculated serum osmolality os usually 280-310 mOsm/kg in healthy animals. Hetastarch and Vetstarch are colloids composed of high molecular weight starches in saline, they are isotonic. Normosol-R and Lactated ringers are a balanced isotonic crystalloids, at 296 and 272 most/L, respectively. %% Dextrose in 0.9% saline is an isotonic crystalloid solution (dextrose is quickly metabolized, leaving 0.9% saline which is isotonic). Normosol-M is actually somewhat hypertonic at 364mosm/L. It is used only for maintenance. It has low levels of sodium and chloride and more potassium than replacement type fluids.

How well did you know this?
1
Not at all
2
3
4
5
Perfectly
43
Q

A 1-year-old female guinea pig is presented with round patches of scaly alopecia on the tip of the nose, ears, periocular areas and forehead. The affected areas do not appear to be pruritic.

Skin scrapings from the periphery of lesions and associated broken hairs demonstrate hyphae and arthrospores on direct microscopic examination.

Which choice is the most likely diagnosis?

A - Cheyletiellosis
B - Dermatophytosis
C - Demodicosis
D - Trombiculosis
E - Notoedric mange
A

b)
This is a typical presentation of ringworm (trichophyton mentagrophytes var mentagrophytes). As with most species ringworm in immunocompetent animals is usually self-limiting with good husbandry and sanitation. Commun: up to 15% of clinically normal guinea pigs harbour T. mentagophytes. Young animal are more susceptible. Overcrowding, poor sanitation and other stressors predispose. Treatment (systemic antifungals, +/- topical shampoos) will speed resolution of infection and minimize chances of infecting other animal or people, but it is somewhat labor intensive and long-term (4-8 weeks). Systemic Rx: itraconazole (10 mg/kg/day, PO) r terbinafine (30-40mg/kg/day, PO) for 4-8 week. If using a topical Rx: Enilconazole (0,2% at a dilution of 1:70) or miconazole shampoo (+/- chlorhexidine), once or twice/week. Decontaminating the environment every 14 days is also important for control: can use enilconazole (0.2%) or concentrated chlorine laundry bleach (1:10) solutions.

How well did you know this?
1
Not at all
2
3
4
5
Perfectly
44
Q

A newborn foal is examined two hours after birth.

The foal has a heart rate of 100 bpm [N=100-120], and a continuous murmur loudest on the left side.

Which one of the following choices is the most likely explanation for these findings?

A - Incipient septicemia
B - Hypoxic ischemic encephalopathy
C - Foal dysmaturity
D - Normal in a neonatal foal
E - Ventricular septal defect
A

d)
These findings are normal in a neonatal foal. The heart rate ad a finding of a continuous murmur on the left side (due to slight opening of the ductus arterioles, which usually closes within 4-5 days) ar normal in newborn foals. Persistent patent ductus arterioles is rare in horses.

How well did you know this?
1
Not at all
2
3
4
5
Perfectly
45
Q

Fractures of the proximal sesamoid bones in horses are often associated with damage to which structure?

A - Suspensory ligament
B - Superior check ligament
C - Deep digital flexor tendon
D - Superficial digital flexor tendon
E - Impar ligament
A

a)
The suspensory ligament is most likely to be damaged with fracture of the proximal sesamoids due to its insertion onto these bones. Proximal sesamoid fractures are relatively common, caused by overextension. The prognosis for return to soundness is often predicated on the extent of the damage to the suspensory apparatus problems are the number one cause of racetrack deaths.

How well did you know this?
1
Not at all
2
3
4
5
Perfectly
46
Q

A stray dog is presented after being hit by a car.

The dog has hypotonic forelimbs and spastic paresis in the hinds.

All four limbs have proprioceptive deficits and sensation loss-signs are worse in the forelimbs.

Where is the lesion?

A - Cranial cervical: C1-C5
B - Cannot say without cutaneous trunci reflex results
C - Lumbosacral L4-S3
D - Thoracolumbar T3-L3
E - Cervicothoracic: C6-T2
A

e)
Cervicothoracic: C6-T2
Weak, hypotonic (lower motor neurone-LMN) forelimbs and spastic paresis (upper motor neuron-UMN) hindlimb are signs of a cervicothoracic (C6-T2) lesion. May see worse signs in fores than hinds.
Note: This is the opposite presentation as Schiff-sherrington syndrome i.e.: severe acute spinal cord trauma in the region of T3-L3, where in lateral recumbency, the thoracic limbs are rigid and extended and the pelvic limbs appear flaccid in comparison. However, pelvic limb reflexes are normal to increased, as would be expected with an upper motor neurone lesion. Can localize T3-L3 lesions by checking cutaneous trunk reflex- the lesion is usually 1-2 vertebrae cranial to where the reflex disappears. With C1-C5 would expect UMN signs in all 4 limbs, usually worse un hinds. With a T3-L3, would see UMN hind limb signs and normal forelimbs.

How well did you know this?
1
Not at all
2
3
4
5
Perfectly
47
Q

Which one of the following choices is the mechanism of action of omeprazole?

A - Cyclooxygenase blocker
B - Beta-adrenergic receptor agonist
C - Synthetic prostaglandin E1 analog
D - H2-receptor antagonist
E - Proton pump inhibitor
A

e)
Omeprazole is a proton pump inhibitor that decreases gastric acid secretion. Omeprazole inhibits the sodium/potassium proton pump at the luminal surface of parietal cells. Parietal cells normally secrete hydrogen ions into the stomach, a key component of acidic HCL. Other drugs that decrease gastric acid secretion include the H2-receptor antagonists cimetidine, ranitidine, and famotidine and a synthetic prostaglandin E1 analog called “misoprostol”. Carprofen, etodolac, deracoxib, meloxicam and firocoxib are all non steroidal anti-inflammatory drugs (NSAIDS) that may cause gastric acid secretion.

How well did you know this?
1
Not at all
2
3
4
5
Perfectly
48
Q

What distinguishes lymphoma from leukemia?

A - Lymphoma is benign
B - Leukemia is derived only from myeloid cells
C - Leukemia is the precursor to lymphoma
D - Lymphoma occurs only in dogs
E - Lymphoma originates in solid tissues

A

e)
Lymphomia originates in solid tissues.
In general, lymphoma and leukaemia both arise from malignant transformation of lymphoid cells and are named based on tutor location. Lymphoma= solid organs and bone marrow. Leukemia= blood circulation and bone marrow.
Leukemias can also arise for myeloid cells (monocytes, basophils, eosiphils, megakaryocytic, erythrocytic precursor cells). Lymphoma is a tutor of lymphocytes that originates mainly in solid lymphoid tissues including the bone marrow, thymus, lymph nodes, and spleen. In addition, common extra nodal sites include the skin, eye, CNS, testis, and bone. Lymphomas are further distinguished according to their location (multi centric, cutaneous, CNS, GI). Leukemia is a malignant neoplastic disease of the WBC, RBC or platelet precursors with neoplastic cells in the bone marrow and peripheral blood. Leukemias are further distinguished according to their behaviour (acute or chronic) and their precursor cell (lymphoid or myeloid). So you will see names like Acute Lymphocytic Leukemia (ALL) or Chronic Myeloid Leukemia (CML). In cats, FeLV infection is an important risk factor for both leukaemia and lymphoma. FIV infected cats have a higher than expected incidence of FeLV-negative lymphomas.

How well did you know this?
1
Not at all
2
3
4
5
Perfectly
49
Q

Fumonisin intoxication is primarily associated with which clinical signs?

A - Leukoencephalomalacia, hypertension
B - Salivation, retching
C - Estrogenism, vulvovaginitis
D - Vomiting, immunosuppression
E - Terminal necrosis of extremities, gangrene
A

a)
Leukoencephalomacia and hypertension.
Fumonisin is another Fusarium app. mycotoxin associated with moldy corn. In equids, look for CNS disease (equine leukoencephalomalacia). In pigs, see hypertension and pulmonary deem (porcine pulmonary edema- PPE). Think of reproductive dysfunction (estrogenism, vulvovaginitis) with zearalone, the only known mycotoxin with primarily estrogenic effects. Slaframine toxicoses causes profuse salivation, primarily in horses and occasionally in cattle. due to the fungus Rhizoctomia leguminocola (black patch disease) on red clover (Trifolium pratense) especially in wet, cool years. Trichothecenes are a group of related cytotoxic mycotoxins associated with many fungi. Think of vomitoxin (and vomiting) and also of immunosuppression. Refusal to eat contaminated feed is a typical sign, due to taste aversion. Macrocyclic trichothecen-related diseases have several specific names, including the best known, staphybotryotoxicosis.

How well did you know this?
1
Not at all
2
3
4
5
Perfectly
50
Q

A lion is presented with a tooth fracture of an upper left premolar, exposing the pulp.

Dental radiographs are normal and no periodontal disease is present.

Which one of the following choices is the best treatment?

A - Wait until the exposure fills in then re-radiograph
B - Extraction
C - Do nothing
D - Root canal therapy
E - Restore the tooth with light cured resin

A

d)
The fracture has left the pulp exposed, requiring endodontic treatment. Extraction is not necessary if the radiographs and the peridontal tissue are normal

How well did you know this?
1
Not at all
2
3
4
5
Perfectly
51
Q

A flock of chickens from a poultry operation is presented with an outbreak which began two days ago with the unexpected discovery of 20 dead birds.

Since then, illness has affected about 25% of the flock.

Birds are depressed and anorexic, many with mucoid discharge from the beak, ruffled feathers, diarrhea and increased respiratory rate.

Two birds have torticollis, and several chickens with swollen sternal bursae, wattles, joints, tendon sheaths, and footpads.

What is the clinical diagnosis?

A - Infectious bronchitis
B - Fowl cholera
C - Infectious coryza
D - Infectious laryngotracheitis
E - Newcastle disease
A

b)
This is fowl cholera, caused by Pasteurella multocida. Think sudden onset septicaemia. Signs vary greatly. In acute fowl cholera, dead birds are first indiction of disease. May see clinical picture described above. With chronic cases, see localized infections: sternal bursar, wattles, joints, tendon sheaths, footpads swell with fibrinosuppurative exudate. See torticolis if meninges, middle ear, cranial bones infected.
With infectious coryza think acute respiratory disease with nasal discharge, sneezing, and swelling under the eyes, cause by Avibacterium (haemophilus) paragallinarum.
With infectious laryngotracheitis (ILT) look for gasping, coughing, blood stained beaks, blood occluding trachea on necropsy. In most states ILT is reportable.
Infectious bronchitis is characterized by respiratory signs, decreased egg production and poor egg quality. Classically may see” wrinkled eggs” with infectious bronchitis.
Newcastle disease is characterized primarily by respiratory signs. Severe forms include depression, neurologic signs or diarrhea. Look fo GI hemorrhage with most severe form, viscerotrpic velogenis newcastle disease (VVND), which is reportable!

How well did you know this?
1
Not at all
2
3
4
5
Perfectly
52
Q

Which one of the following choices is the most appropriate next step after a 3-year-old stray cat tests positive for Feline Immunodeficiency Virus (FIV) on a routine FIV ELISA screening test?

A - Recheck serum by FIV complement fixation test
B - Do a Rivalta test to rule out feline coronavirus-related interference
C - Treat with glucocorticoids
D - Perform for a Western blot FIV test
E - Euthanize

A

d)
Perform a Western blot FIV test.
The feline immunodeficiency virus (FIV) ELISA is the standard screening test to determine if the cat has circulating antibody to FIV. A western blot antibody test for FIV is the standard confirmatory antibody test. However, the western blot also cannot differentiate antibodies caused by infection from those caused by vaccination. Note- new PCR tests show promise for confirming FIV infection regardless of vacicnation status, but are not yet the standard of clinical practice. PCR was not among the answer choices in this question to avoid confusion, and mimic the real test. On the real exam, you should not expect to see equivocal, controversial or cutting-edge new treatments or diagnostic tests.
A western blot test is especially important in areas with low FIV prevalence, where the risk of false positive FIV ELISA is higher.
Remember that cats vaccinated for FIV will test positive for FIV antibodies by FIV eLISA and western blot. Kittens up to 6 months of age born to seropositive queens can also be seropositive, even though they are not infected, due to persistence maternal antibodies. Seropositive kittens should be retested at grease than 6 months of age. This is different form feline leukemia virus (FeLV) testing because the FeLV eLISA and IFA measure antigen, not antibodies, so FeLV vaccination does not interfere with testing

How well did you know this?
1
Not at all
2
3
4
5
Perfectly
53
Q

You are helping a local goat dairy owner interpret the results of semi-annual ELISA serological testing of her herd for caprine arthritis encephalitis (CAE).

One 2-month-old kid is CAE-positive by ELISA. Another 2-month-old kid is CAE-negative by ELISA.

How do you interpret these results?

A - Cannot tell if either kid is truly CAE positive or negative
B - CAE-positive kid has CAE, CAE-negative kid does not have CAE
C - Both kids are infected with CAE
D - CAE-positive kid has CAE, cannot say if CAE-negative kid is infected or not
E - Cannot say if CAE-positive kid has CAE, CAE-negative kid is not infected

A

a)
You cannot tell if either kid is truly CAE positive or negative yet. Should probably retest both after 90 days, or at next semiannual herd testing. A positive ELISA test for caprine arthritis encephalitis (CAE) in a kid less than 90 days old may reflect maternal antibody or a false positive. A negative test in a kid less than 90 days old does not rule out CAE (though most kids infected at birth form colostrum develop a measurable antibody response by 4-10 weeks after infection). Note lots of qualifiers about CAE testing: A positive test in an adult goat implies infection, but does not confirm that clinical signs were caused by CAE virus. Negative tests do not rule out CAE infection. Definitive Dx is to find characteristic lesions by biopsy/necropsy. Virus isolation can substantiate Dx, but may not be practical on herd basis.

How well did you know this?
1
Not at all
2
3
4
5
Perfectly
54
Q

A 9-year-old German shepherd is presented with unchecked bleeding from a cut on the gums above the right canine tooth. The owner relates that the dog has lost weight and had an episode of collapse 3 days ago, but he recovered.

On physical exam, the gums are pale with petechiae and ecchymotic hemorrhages. There is tachycardia and a palpable cranial abdominal mass.

A coagulation profile shows the following:

Thrombocytes= 82,533 per microliter..[N=200,000-900,000]
Buccal mucosal bleeding time (BMBT), increased
Activated partial thromboplastin time (aPTT), increased
Prothrombin time (PT), increased
Thrombin time (TT), increased
Fibrin degradation products (FDPs), increased

What disorder of coagulation best fits this pattern?

A - Von Willebrand's disease
B - Idiopathic thrombocytopenia
C - Hepatic insufficiency
D - Anticoagulant rodenticide toxicity
E - Disseminated intravascular coagulation
A

e)
A lab pattern of low platelets, increased bleeding time and across the board increases in aPTT, PT, TT and FDP tests suggests disseminated intravascular coagulation (DIC). DIC is not a disease in its own right- it is a complex hemostatic defect characterized by enhanced coagulation and fibrinolysis, secondary to other diseases. Firbinolysis and depletion of clotting factors leads to hemorrhage. Many, many diseases, all of them bad, can precipitate DIC. This case presentation (pale, old German shepherd with Hx of collapse, bleeding and an abdominal mass) suggests hemangiosarcoma.
Remember you H disease associated with DIC: heartworm, heart failure, hemolytic anemia, hemangiosarcoma, hemorrhagic gastroenteritis, hepatic disease, especially hepatic lipidosis in cats.
Gastrci dilatation-volvulus, mammary gland carcinoma and pancreatitis can also lead to DIC.

How well did you know this?
1
Not at all
2
3
4
5
Perfectly
55
Q

Roughly 10% of the dry sows in a herd of pigs have poor appetite, fever, and are coughing.

There have been several late-term abortions and an increase in returns post-service.

Some of the farrowing sows have poor appetite and thirst.

Several have mastitis and/or agalactia and are farrowing 23 days early.

Some piglets born alive have diarrhea, and some in the litter were born dead and mummified.

Theres been more respiratory disease in the piglets recently and the farmer has heard a funny thumping noise when the most severely affected piglets breath.

The farmer noticed that several of the sows have transiently had blue ears.

Based on the presumptive diagnosis, if this disease becomes endemic in the herd, which of the following is a likely sequela?

A - Necrotic ear syndrome in piglets
B - Enzootic pneumonia in grower/finisher units
C - Chronic diarrhea in breeding sows
D - Neurologic syndromes in breeding boars
E - Diamond-shaped, erythematous skin lesions in most pigs

A

b)
Enzootic pneumonia in grower/finisher units is a likely sequela of porcine reproductive and respiratory syndrome (PRRS). This history ans clinical presentation are consistent wit PRRS. It is common for the grower/finisher units to have increased severity of enzootic pneumonia once the infection has spread to all parts of the herd. PRRS is caused by a highly contagious arterivirus that is spread via fomites, aerosolization, insects, and silent shedders. There are two typical phases. First, reproductive failures as described in this herd and second, post weaning respiratory diseases. The initial outbreak lasts about 1-4 months, but the disease can become chronic and endemic. PRRS is the most economically significant disease affecting US swine production today. Dx: virus detection and serology on oral fluid samples. Rx: none, prevention is key. Know the status of replacement gilts and boars, isolate incoming stock. Nursery depopulation has been used 12-18 months after the initial outbreak when only the growers/finishers are still affected. Other effective methods: whole herd depopulation-repopulation, test and removal, and herd closure.

How well did you know this?
1
Not at all
2
3
4
5
Perfectly
56
Q

This 11-year-old spayed female cat is presented with generalized weakness and is unable to lift her head.

The owners have noticed her at the water bowl more often and she seems to be urinating more in the last month.

Which blood chemistry test would be most likely to diagnose cause of her weakness?

A - Sodium
B - Potassium
C - Chloride
D - Calcium
E - Bicarbonate
A

b)
Potassium. This is a classic presentation of a cat with hypokalemic myopathy which is often due to potassium loss from renal disease. Other causes of generalized weakness and neck ventroflexion include polymyositis, organophosphate toxicity, myasthenia gravis

How well did you know this?
1
Not at all
2
3
4
5
Perfectly
57
Q

Several younger sheep in a large herd have died suddenly with a diagnosis of infectious necrotic hepatitis secondary to fluke infestation.

Which choice is the most practical and effective control measure you can take next to prevent future cases?

A - Vaccinate all animals with Clostridium novyi toxoid
B - High-dose penicillin for clinical cases
C - Clostridium hemolyticum bacterin for animals under 3 years
D - Clorsulon antihelmintic for whole herd
E - Treat ponds with copper sulfate molluscicide against lymnaeid snails

A

a)
Active immunization with Clostridium novae toxoid before the late summer is the most effective way to control and prevent infectious necrotic hepatitis (Black disease) in sheep. Because vaccinated sheep have long-term immunity after only one shot, only new introductions to the flock (lambs and sheep brought in from other areas) need to be vaccinated. Reducing the number of snails (intermediate hosts for fluke cercaria) with molluscicides or by fencing off wet areas may not be practical due to expense or amount of pasture lost. Likewise, use of flukicides is complicated but the need for careful timing and long withdrawal times for meat and milk.

How well did you know this?
1
Not at all
2
3
4
5
Perfectly
58
Q

A rabbit is presented with inflamed and chapped membranes of the anus and genital region.

The genital area is scalded and raw, with brownish crusts and purulent exudate.

What two conditions top the differential diagnosis list?

A - Tularemia, cystitis
B - Treponematosis, hutch burn
C - Myxomatosis, moist dermatitis
D - Glomerulonephritis, coccidiosis
E - Pasteurellosis, ulcerative pododermatitis
A

b)
Treponematosis (rabbit syphilis, vent disease) and hutch burn (urine burn) resemble each other and are often confused.
Treponematosis (rabbit syphylis, vent disease) is a venereal disease of rabbits caused by Treponema paralysis cuniculi. Affects the genitalia, may affect eyes and nose.
Hutch burn is caused by wet and dirty floors, affecting the anus and genitalia.
Remember that caudal equine neuritis (polyneuritis equipment) in horses may present with utile scald on the thighs. Other signs include a weak tail, hypotonic anus, urine dribbling and focal retention. There may be a history of rubbing or chewing the the tail head.

How well did you know this?
1
Not at all
2
3
4
5
Perfectly
59
Q

A 4 month old calf from a small beef herd in Louisiana presents with a fracture of the right metatarsus.

He was normal last night.The owner found the calf this morning after rounding up the herd for deworming.

This is the 5th calf this season with fractured bones.

One had a vertebral fracture, two calves had humeral fractures, and one other had a cannon bone fracture.

Which one of the following choices is the most likely problem in this herd?

A - Low dietary copper
B - High dietary phosphorus
C - Hypovitaminosis D
D - Low dietary protein
E - High dietary calcium
A

a)
Low dietary copper levels causes bones of growing animals to be brittle because they do not develop the normal trabecular structure. Higher fractures rates are seen in Cu deficient than in Cu replete animal. Pathologic fractures are more common during periods of exertion, such as the morning round up. Rough handling exacerbates the number of fractures. Treatment off Cu deficiency includes copper glycerinate injections, and feeding of mineral mixes with Cu. Copper boluses can also be given. In areas with high levels of molybdenum (Mb) in the soil, Cu levels are often low. Mb interferes with absorption of Cu, hence Mb excess results in Cu deficiency. Pathologic fractures can be seen with hypovitaminoses D growing cattle, but this is unlikely in beef on pasture in the southern US. Calcium deficient calves may also develop a similar syndrome of pathologic fractures.

How well did you know this?
1
Not at all
2
3
4
5
Perfectly
60
Q

A middle-aged female spayed dachshund presented with polydipsia and polyuria but otherwise healthy with no fever. Examination reveals a distended abdomen with thin skin and alopecia.

Which one of the following choices is the most likely suspected diagnosis?

A - Hepatic insufficiency
B - Stump pyometra
C - Hyperadrenocorticism
D - Urinary tract infection
E - Hydronephrosis
A
c)
abdominal distension (potbelly), polyuria, polydipsia (PU-PD) and cuteness atrophy are common signs of hyperadrenocorticism. About 85% of hyperadrenocorticism cases are pituitary dependent. Stump promettra can also cause PU-PD, but the dog usually is ill and does not have the cutaneous atrophy.
How well did you know this?
1
Not at all
2
3
4
5
Perfectly
61
Q

Damage to the left recurrent laryngeal nerve is associated with “roaring” in horses.

The left recurrent laryngeal nerve is a branch off of which cranial nerve?

A - Glossopharyngeal (CN 9)
B - Facial (CN 7)
C - Trigeminal (CN 5)
D - Hypoglossal (CN 12)
E - Vagus (CN 10)
A

e)
Damage to the recurrent laryngeal nerve ( a branch of the vagus (CN 10)) causes laryngeal hemiplegia- a paralysis of the abductor muscle (dorsal cricoarytenoid muscle) controlling the glottic cleft in the larynx. This allows the vocal fold (usually left side) to evert into the lumen of the larynx, obstructing airflow, leading to a roaring sound, and most importantly, slowing the horse

How well did you know this?
1
Not at all
2
3
4
5
Perfectly
62
Q

A pig farm reports a problem in weaning-age piglets.

The affected piglets are depressed and anorexic with slight fevers.

A few of the more severely-affected piglets have an exudative epidermitis affecting the face, limbs, and ventral abdomen.

What should be recommended?

A - High-dose antibiotics for sick piglets
B - Cull affected animals
C - Vaccinate herd with modified live vaccine
D - Report to state vet
E - Topical coumaphos/disinfectant combination dip

A

a)
Use high-dose antibiotics for sick piglets. This is exudative dermatitis (“greasy pig disease”), caused by Staph hyicus. Responds to most antibiotics (pen, amoxicillin, TMS, erythromycin, lincomycin, tyrosine, Ans, ceps) at high dose 7-10 days, plus topical antiseptic. In severe outbreaks, antibiotics for contact pigs recommended for a few ays. Disinfection, good environnemental hygiene important. In chronically affected herds, autogenous bacteria’s have been used with some success to decrease incidence.

How well did you know this?
1
Not at all
2
3
4
5
Perfectly
63
Q

Two bucket-fed veal calves are depressed and stunted and have sticky feces coating their hindquarters.

Fluid-splashing sounds are audible during auscultation over the left flank when a calf is drinking.

Passage of a stomach tube obtains a rancid-smelling liquid with a pH of 5.2 [N=5.9-6.2]

Which one of the following choices is the most appropriate treatment?

A - Remove fermented rumen contents, flush with saline
B - Wean affected calves; Shift to bottle-feeding for well calves
C - Vitamin E/Selenium injections, plus dietary supplements
D - Cull affected calves
E - Inoculate rumen fluid from a healthy cow into calf by stomach tube

A

a)
Remove fermented rumen contents, flush with saline
This is the clinical picture of a luminal drinker calf with chronic indigestion from milk deposited directly into the rumen. Treat by removing the fermented material and flushing the rumen with saline. Attempt to induce gastric/reticular groove closure during feeding by inducing vigorous sucking activity with a finger before feeding milk. If calves relapse, consider weaning. The problem occurs due to failure of the gastric groove reflex which normally shunts milk directly into the abomasum (bypassing the rumen). Most often seen in bucket fed claves who gulp their milk.

How well did you know this?
1
Not at all
2
3
4
5
Perfectly
64
Q

A middle-aged, depressed, coughing, exercise-intolerant Doberman pinscher is presented.

The dog has a rapid and irregular heart rate.

What disease is at the top of the differential diagnosis list?

A - Dilated cardiomyopathy
B - Mitral regurgitation
C - Chronic bronchitis
D - Wolf Parkinson White syndrome
E - Tricuspid insufficiency
A

a)
This is a classic presentation for dilated cardiomyopathy. Prognosis is guarded- with Rx may live for 6 months to 2 years. Px especially poor for dobermans. Rx varies, but foundation includes the “3 D’s” – Diet (low salt), vasoDilatation (enalapril), Diuretics (furosemide), +/- Digoxin (+ionotrope to strengthen contraction). Note that dobermans can be sensitive to digitalis, so may need lower doses. So, remember your “DCM D’s”: depressed dilated dogies need diet, dilators, diuretics (and maybe digoxin, but overdose is doom).

How well did you know this?
1
Not at all
2
3
4
5
Perfectly
65
Q

The parents of a ten-year-old boy present his pet box turtle with a three-week history of lethargy and anorexia.

On physical exam, there is bilateral blepharedema (swollen eyes), as well as ocular and nasal discharge.

Which one of the following choices is the most likely diagnosis?

A - Inclusion body disease
B - Septicemic cutaneous ulcerative disease (SCUD)
C - Vitamin A deficiency
D - Herpesvirus
E - West Nile virus (WNV)
A

c)
This is the clinical picture of vitamin A deficiency, a problem primarily seen in Turtles, but also in alligators and other reptiles. Look for swollen eyes, discharge from nose/mouth/eyes and ear infections. Vitamin A deficiency can pridespose to a bacterial pneumonia. Like other key reptile issues (metabolic bone disease for example), the problem is basically nutritional and preventable.
Inclusion body disease is a viral problem of boa constrictors and some pythons. Septicemic cutaneous ulcerative disease (SCUD) is a shell disease (Citrobacter freundii). Look for pitted shells.
Herpesvirus causes oral mucosa necrosis, anorexia, regurgitation, oral, ocular discharge, but discharge from the nose and swollen eyes should put vitamin A deficiency, not herpes, at the top of your DDX.
West nile virus (WNV) has been diagnosed in farmed alligators with multi organ necrosis, heterophilic granulomas, heterophilic perivasculitis, and lymphoplasmacytic mengiongoencephalitis. Very high WNV viremia tigers have been reported in different reptiles, suggesting that they may play a role as amplification hosts for the virus.
Zoonotic note: remember salmonella when you think of hildren with pet turtles or other pet reptiles

How well did you know this?
1
Not at all
2
3
4
5
Perfectly
66
Q

In September, a 15-year-old Quarterhorse mare is presented with a five-day history of left head tilt, facial paralysis, depression, and stumbling.

The horse is ataxic and knuckles on both forelegs, worse on the left.

There is muscle atrophy of the left pectorals and right hindquarters and strips of localized spontaneous sweating over the left trunk.

A serum:CSF ratio is consistent with a diagnosis of Sarcocystis neurona.

What prognosis is given to the owner if she elects to treat the horse?

A - Only 25% of treated horses improve in their neurologic state
B - 50% of treated horses recover completely
C - Prognosis is good if treated with doxycycline
D - At least 60% of treated horses improve
E - Prognosis is grave

A

d)
At least 60% of treated horses improve. The prognosis is guarded to faire for treatment of equine protozoal myelencephalitis. Less than 25% recover completely, but at least 60% of treated horses improve. Affected animal typically improve by 1-2 ataxia grades (on the mayhem ataxia scale of 1-5). The earlier treatment is started, the better the outcome. !–20% of horses relapse within 2 years of initial diagnosis.

How well did you know this?
1
Not at all
2
3
4
5
Perfectly
67
Q

What condition is associated with rectal stricture in pigs?

A - Rotavirus
B - Intestinal spirochetosis
C - Salmonella typhimurium 
D - Intussusception
E - Coccidiosis
A

c)
Salmonella typhimurium os associated with rectal stricture in growing pigs. Caused by an ulcerativeproctitis that damages rectal tissue. Can see large numbers of cases. Can see sporadic rectal strictures as a sequelae to rectal prolapse.
Intestinal spirochétoses is a post-weaning diarrhea seen in the absence of Brachyspira (serpulinaj) hyodusenteria (swine dysentery), but similar presentation to it. This syndrome is being recognized more frenquetly worldwide.

How well did you know this?
1
Not at all
2
3
4
5
Perfectly
68
Q

A ten-year-old male intact Doberman mix dog is presented with one testicle that is grossly enlarged and nonpainful and the other atrophied.

What condition(s) are expected to be seen in association with this presentation?

A - Benign prostatic hypertrophy, constipation
B - Hypothyroidism
C - Increased aggression, territoriality
D - Feminization syndrome
E - Penile hypertrophy, hirsutism
A

d) Feminization syndrome.
Nonpainful unilateral testicular selling in an old dog should make you think if neoplasia, especially an estrogen-secreting sertoli cell tumor, which can cause feminization syndrome. See gynécomastie, penile atrophy, pendulous prepuce, attraction of other males, blood dycrasias, +/- bone marrow depression (via high estrogen), bilateral alopecia. Remember that cryptorchidism increases risk of sertoli cell tumor. About 25%-29% of sertolis develop feminization syndrome and 70% of intra-abdominal testicular tutors, regardless of type, cause feminization syndrome. May see prostate hypertrophy with sertoli, but it is due to squamous metaplasia, not the classic (and common) benign prostatic hypertrophy (BPH). Hypothyrodoism and BPH are common older dog conditions, but not associated with unilateral testicular hypertrophy. Remember that sex hormones are also secreted but the zona reticulates of adrenal cortex and can see feminization or virwlism with excess secretion.

How well did you know this?
1
Not at all
2
3
4
5
Perfectly
69
Q

Which disease of rabbits is reportable?

A - Rabbit (Shope) fibroma virus
B - Clostridium spiroforme enterotoxemia
C - Encephalitozoonosis
D - Treponematosis
E - Francisella tularensis
A

e)
Tularemia is a disease primarily of wild rabbits caused but
Francisella tularensis. Upt to 90% of human tularaemia cases are linked to wild lagomorph exposure. Tularemia is a Category A bioterrorism agent. Reportable.
Treponematosis (rabbits syphilis, vent disease) is a venereal disease of rabbits caused but Treponema parlais cuniculiculture.
Encephalitozoonis is a protozoal disease caused by Encephalitozoon (Nosema) funiculi. Typically does not cause illness in rabbits, but is an emerging disease of immunosuppressed humans. Myxomatosis ( “big head”) is a fatal disease of domesticated rabbits caused by myxoma virus, characterized by mutinous skin lesions or “myxomas”. According to USDA APHIS, myxomatosis is also reportable.

How well did you know this?
1
Not at all
2
3
4
5
Perfectly
70
Q

At what day post breeding (the earliest) can an experienced practitioner feel the fetal membrane slip of the chorioallantoic membrane of a cow?

A - 20-25 days
B - 40-45 days
C - 30-35 days
D - 10-15 days
E - 45-50 days
A
c)
An experienced practitioner can reliably feel the "membrane slip"of bovine chorioallantoic membranes rectally  at 30-35 days. Elle placentomes around 70-75 days; feel fremitus ( vibration in uterine artery of the gravid uterine horn) between 90-120 days. One way to remember these is that the earliest occurs in reverse alphabetical order i.e.: 
Slip 30-35 days
Placentomes 70-75 days
Fremitus 90-120 days
71
Q

A three-year-old white rat is presented with yellowing fur and a brownish, granular sebaceous secretion at the base of affected hair shafts.

What should the owner be told?

A - Moisture-related dermatitis, cage needs more frequent cleaning
B - Staph dermatitis, responsive to antiseptic shampoo
C - Common manifestation of chorioptic mange
D - Normal age-related change
E - Probable ringworm, resolves spontaneously with sunlight exposure

A

d)
These are normal age related changes. Rat normally live two to three years, though some may live to five years of age. With age, the hair coat of white rats normally develop yellowing fur. They may also show a brownish, granular sebaceous secretion at the base of affected hair shafts which an owner may mistake for a skin parasite

72
Q

A three-day-old female alpaca (cria) is presented in respiratory distress.

The cria’s cheeks flare noticeably during inspiration, and the distress is more pronounced during nursing, when the animal gasps and inhales milk.

What is the most likely diagnosis?

A - Lung lobe torsion
B - Wry face
C - Diaphragmatic hernia
D - Choanal atresia
E - Mitral stenosis
A

d)
Choanal stressa is one of the most common congenital problems of South american camelids. Choanal atresia occurs when the caudal nares (choanes) fail to open during embryologic development. Can be unilateral or bilateral and may cause complete or partial blockage.

73
Q

What acid-base abnormality would be most likely in a dairy cow with a displaced abomasum?

A - Metabolic alkalosis
B - Metabolic acidosis
C - Depends on severity of displacement
D - Respiratory alkalosis
E - Respiratory acidosis
A

a)
Think of hypochloremic metabolic alkalosis due to hydrochloric acid (HCl) sequestration in the abomasum of a cow with a displaced abomasum. Abomasal hypo motility, ongoing Hal secretion into the abomasum, and partial abomasa outflow obstruction all contribute. Metabolic alkalosis can also be due to HCl loss in mono gastric animals who vomit. In a similar way, metabolic acidosis can be due to HCO3 loss in saliva if animal cannot swallow, or from diarrhea.

74
Q

What recommendation should be given to the owner of a mare that has had Caslick’s vulvoplasty surgery?

A - Mare has a higher than normal risk for metritis
B - Mare has a decreased risk for perineal laceration
C - Mare should not be bred by a stallion
D - Mare should not be bred
E - Mare will need an episiotomy before foaling

A

e)
The mare will need an episiotomy ( a surgically planned incision on the perineum and the posterior aginal wall) prior to foaling. Sometimes an episiotomy may be needed prior to breeding by a stallion if the remaining vulvar cleft is too small to permit intromission. A Caslick operation (vulvoplasty, a superficial form of episioplasty) is used to treat pneumovagina in horses to prevent genital infections and infertility.

75
Q

A person calls to say she adopted a friend’s dog and requests a copy of the dog’s medical record and radiographs. The former owner is a client at your clinic.

Which one of the following choices is the most appropriate action to take?

A - Require original owner consent or court order to release information
B - Give caller a copy of medical record only
C - Give caller a copy of medical record and radiographs
D - After physical exam, start a new medical record documenting previous issues
E - Give a summarized record with personal identifiers (vet, owner names) blocked out

A

a)
According to the AVMA’s principles of veterinary medical ethics, “ethically, the information within veterinary record is considered privileged and confidential. It must not be released except by court order or consent of the owner of the patient. Veterinarians should secure a written release to document that request.”

76
Q

A two-day-old pup has a GGT level of 500 times the adult upper reference range.

Which one of the following choices is the most likely interpretation?

A - Associated with neonatal cholestasis
B - Related to bone development
C - Caused by traumatic placental detachment
D - Due to colostrum absorption
E - Suggestive of a congenital liver shunt

A

d)
Very high GGT levels are seen with colostrum absorption. In newborn pups, lambs, and claves, an increase in GGT up to a 1000 times is normal when they receive high levels of colostrum from the dam. High levels of GGT are produced in the mammary epithelium during lactation. When evaluating bloodworm, reference ranges specific for the age of the patient should be utilized.

77
Q

An outbreak of diarrheal disease of piglets has occurred which affected the healthiest animals in the herd, one to two weeks after weaning.

Some affected piglets had no signs except peracute death.

Other affected piglets exhibit diarrhea, ataxia, paralysis, and recumbency.

What condition is at the top of the differential diagnosis list?

A - Clostridium perfringens type C enteritis
B - Hemagglutinating encephalomyelitis virus (HEV)
C - Porcine proliferative enteritis
D - Epidemic transmissible gastroenteritis (TGE)
E - Edema disease

A

e)
Edema disease is caused by Shiga toxin-producing E. coli (STEC). Look for severe acute illness ranging from parachute death with no signs to CNS involvement with ataxia, paralysis, and recumbency in healthiest pigs one to two weeks after weaning.
Hemagglutinating encephalomyelitis virus (HEV), is almost exclusive to piglets less than four weeks old. Two clinical presentations: vomiting and wasting disease (VWD) and encephalitic. Clostridium perfringens type C enteritis, also called enterotoxemia in other animals, is characterized by a hemorrhagic diarrhea in one to three day old piglets. Porcine proliferative enteritis i principally a diarrheal disease of growing finishing (40 to 80 lbs) pigs and young breeding pigs. Epidemic transmissible gastroenteritis (TGE) in non-immune pig herd characterized by high morbidity and high mortality in piglets less than one week old.

78
Q

A herd of ewes has had an abortion storm in which 80% of animals are affected.
All stages of gestation are affected, and lambs that are born are very weak.
On the placentas, cotyledons have white foci of necrosis.
Which one of the following choices is the primary differential diagnosis?

A - Listeria monocytogenes 
B - Brucella 
C - Chlamydiosis
D - Histophilus ovis 
E - Toxoplasma gondii
A

e)
Toxoplasma gondii infection can cause severe abortion storms in sheep. Clinical signs pathognomonic of T. gonds are bright to dark red cotyledons with white foci of necrosis. The intercotyledonary areas of placenta are normal.
With Brucella and Histophilus obis the abortion rate is much lower. Abortion caused by Listeria spp. is associated with the feeding of silage.

79
Q

Which one of the following management recommendations is most helpful to reduce the incidence of mare reproductive loss syndrome?

A - Cut down all the black walnut trees on premises
B - Vaccinate mares and stallions against equine coital exanthema
C - Remove the horses from affected pastures
D - Progesterone supplementation between 4-6 months of pregnancy
E - Spray paddocks to kill Habronema muscae larvae

A

c)
Removal of mares from affected pastures is necessary for control of mare reproductive loss syndrome (MRLS). The exact pathogenesis of MRLS is unknown, but exposure to/ingestion of “hirsute” or hairy caterpillars appears to be a risk factor. Early and late-term abortions have been reproduced by oral administration of whole Eastern tent caterpillars (Malacosoma americium) or their exoskeletons. Eastern tent caterpillars were associated with a large outbreak in Kentucky in 2001-2 here’re over 3000 pregnant mares aborted. Other episodes have been seen in Florida (2005) and Australia (2007) with exposure to other animals of hirsute caterpillars. The most current theory - damage to the oral mucosa or intestinal lining by the setae (spines) of the caterpillars introduces bacteria that causes bacteriaand subsequent infection of the fetus and/or placenta. Early embryonic loss, abortion, stillbirths, weak foals, and infected foals are seen. Swollen umbilical cords are also often seen in aborted fetuses. Feeding hay and removal of wild cherry trees, a major food source for caterpillars, is also beneficial in reducing the incidence of MRLS abortion.

80
Q

A five-year-old domestic short-haired spayed female cat is presented with the complaints of anorexia, chronic vomiting, and weight loss.

Physical examination reveals icterus, dehydration, and fever.

Thickened small intestinal loops are appreciated with abdominal palpation, and serum biochemistry shows increased bilirubin and liver enzymes.

The cat is diagnosed with extrahepatic biliary obstruction with accompanying cholangitis and inflammatory bowel disease.

Which one of the following normal anatomic features may predispose cats to this triad of diseases?

A - Narrowing of the duodenum distal to the entry of the common bile and pancreatic ducts
B - Common opening of the pancreatic and common bile ducts into duodenum
C - Enlarged duodenal papilla in comparison to other species
D - Decreased proportion of alpha-smooth muscle actin in muscle fibers of the wall of the gall bladder in cats
E - Decreased diameter and increased branching of the hepatic ducts in comparison to other species

A

b)
In cats, there is a common opening of the pancreatic and common bile ducts into the duodenum. It is thought that this may predispose them to ascending cholangitis and pancreatitis after vomiting associated with inflammatory bowel disease, resulting in extrahepatic biliary obstruction. Other possible aetiologies of EHBO include neoplasia, biliary stricture, duodenal obstruction, diaphragmatic hernia, and parasitic infection. In allocates, there is a lack of bile entry into the intestinal tract, leading to decreased absorption of fat and fat soluble vitamins such as vitamin K, potentially resulting in coagulopathies. Except in transient cases that are related to acute pancreatitis, surgical intervention to relieve the obstruction is required as well as appropriate supportive car, antimicrobial treatment, and vitamin K administration.

81
Q

During examination of a newborn male cria, adhesions between the free end of the penis and the prepuce are evident.

What is the most likely explanation?

A - Normal finding
B - Common congenital malformation of preputial ring, associated with phimosis
C - Secondary to stress in utero, 5th month
D - Failure of separation of preputial diverticulum
E - Sequella of hypospadias; Heritable

A

a)
Normal male crias are born with adhesions between the free end of the penis and the prepuce. AS they sexually mature these adhesions gradually detach, so puberty can be seen clinically when the penis can be completely exteriorized. Castration prior to puberty may result in continued adhesions. In Peru, males are not generally used for breeding until 3 years of age. Male slams reach puberty at approximately 21 months (range 9-31 months). Alpacas mature earlier, around 12 monts of age.

82
Q

A female veterinarian is pregnant and her physician tests her serologically for toxoplasmosis.

IgM is negative.
IgG is positive.

What is the most appropriate interpretation?

A - Mother is safe, baby at risk
B - Mother at risk, baby is safe
C - Both mother and baby are safe
D - Need to re-check in 2 weeks for rising titers
E - Both mother and baby at risk
A

c)
Both mother and baby are safe. The toxoplasmosis organism causes birth defects in a developing fetus if a mother is infected for the first time in her life while pregnant (i.e.: IgM positive while pregnant). Toxoplasmosis is not generally dangerous to immune-competent people and a positive IgG result suggest a previous infection. There are challenges to toxoplasmosis testing in pregnant women (false positives). If a pregnant woman is IgM positive, confirmatory tests must be done.

83
Q

Which breeds are predisposed to mitral regurgitation? (The number one cause of congestive heart failure in dogs).

A - Cavalier King Charles spaniel, Cocker spaniel
B - Wirehaired fox terrier, Border collie
C - Great Dane, Afghan hound
D - Miniature schnauzer, German shepherd
E - Doberman pinscher, Boxer

A

a)
Think of mitral regurgitation/insufficiency on a Cavalier King Charles Spaniel of any age, or in old male Cockers with a Hx of cough, laboured breathing, and exercise intolerance. Remember that chronic obstructive pulmonary disease with fibrosis can cause similar signs-primarily older animals. Here are 4 other big cardio predispositions: If you see an intermittently fainting Boxer, think of Boxer cardiomyopathy. A rapid, irregular heart rate in a middle-aged, depressed, coughing, exercice-intolérant Doberman, suggests dilated cardiomyopathy (DCM). Think of Sick Sinus Syndrome (SSS) when you hear “Fainting Female Miniature Schnauzer”. Also see SSS in dachshunds, cockers, west highland whites. Think Tetralogy of Fallot in a young bulldog, keeshond, wire-haired fox terrier with cyanosis, exercise intolerance. Remember young Miniature Schnauzers and Wirehaired Fox terriers are predisposed to Mega-esophagus.

84
Q

A seven-year old male German shepherd is presented with a history of weakness in the hind limbs, urinary incontinence and recent obsessive chewing around his tail area.

Dorsiflexion of the tail over the back and extension of the hind limbs elicits a painful response.

He does not withdraw each hind leg when a toe is pinched, but bears weight on the hindlimbs. Patellar reflexes are normal.

What is the clinical diagnosis?

A - Wobbler syndrome
B - Diskospondylitis
C - Hip dysplasia
D - Cauda equina syndrome
E - Radiculoneuritis
A

d)
This is a common presentation of Cauda equine syndrome. Look for PAIN in the lumbosacral area (elicited by tail raise, hindlimb extension), LMN hindlimb, especially Sciatic nerve damage at L7-S1 (lack withdrawal), +/- urinary/fecal incontinence, +/- self mutilation of tail, perineum, pelvic limb.
Lesion due to compression of caudal equine at L7-S1 (lumbosacral stenosis). Can be congenital (awn development dorsal arch L7-S1, small dogs, Border collies) or acquired (degenerative changes, big dogs, especially German Sheperd, Rottweiler, Boxer). Wobbler syndrome (aka cervical vertebral instability, (caudal) cervical spindomyelopathu) is a congenital cervical spinal cord disease. Affected dogs are born with vertebral canal stenosis which can worsen later in life due to intervertebral disc disease, vertebral malformation and/or ligamentous hypertrophy. Usually seen at C5-C6 or C6-C7 in Dobies>5 years, Great Danes< 2 years. Diskospondylitis is a good second choice on your DDX of a large middle aged dog presenting with lumbosacral pain. Due to bacterial/fungal infection of intervertebral disk and adjacent vertebral bodies. Look for systemic signs like fever (1/3 of patient), weight loss, anorexia.

85
Q

Foal heat diarrhea is typically associated with which choice?

A - Hemorrhagic enteritis
B - Neutropenia and fever
C - Decreased suckling
D - Alterations in diet
E - Warm and humid weather
A

d)
Alteration in diet
Mild, self-limiting diarrhea in a foal 7-14 days of age is called foal heat diarrhea, because it coincides with the first estruc cycle post-foaling in the dam. Causes of foal heat diarrhea are poorly understood, but are thought to be related to a foal’s tendency to start sampling hay and grain practice coprophagy by 5 to 7 days of age, with consequent alterations in bacterial flora. Foals are active and alert, with a normal appetite and vital signs. Clinical signs such as fever and lethargy, hematochezia or melon, and laboratory findings such as neutropenia are not routinely observed.

86
Q

Which of these statements about pasteurellosis in rabbits is most accurate?

A - 2-week course of antimicrobials is effective in eliminating disease
B - It is a gram-positive, nonmotile coccobacillus
C - Every positive test result indicates a pathologic condition
D - Most common presentation is chronic, insidious arthritis
E - 30-90% of adults in colonies are asymptomatic carriers

A

e)
30-90% of adults in colonies are asymptomatic carriers of Pasteurella multocida so not every positive nasal swab means pathology. Like shipping fever in cattle, pasteurellosis in rabbits is multifactorial: stress (crowding, kindling) can precipitate disease and other organisms can be isolated from clinical cases. (Pseudomona app., Bordetella bronchiseptica, Staph app. and Strep app.). P. multimode is gram-negative, nonmotile coccobacillus. Therapy is difficult and even long-term antibiotics often do not completely eliminate the bacteria.

87
Q

An adopted adult male cat from a shelter with an unknown vaccination history is presented. The cat tests positive for feline leukemia virus (FeLV) infection by IFA.

What should the owner be told?

A - Possible transient FeLV infection
B - Need an ELISA test in 12 weeks to confirm diagnosis
C - Cat may be vaccinated for FeLV
D - Need a Western blot test to confirm diagnosis
E - Strong chance of lifelong FeLV infection

A

e)
97% of cats positive for feline leukaemia virus (FeLV) by IFA remain persistently infected and viremic for life. The FeLV ELISA is more sensitive that the IFA (fewer false negatives, so trust a negative test more), so it is a better general screening test with which to start. The FeLV IFA is more specific than the ELISA. (fewer false positive, so trust a positive test more) so it is a better confirmatory test for any cats with a positive FeLV ELISA.
Three things to remember about FeLV and vaccinations:
1. FeLV ELISA and IFA tests measure antigen, not antibody, so FeLV vaccination does not interfere with testing.
2. Vaccinate FeLV-positive cats yearly against respiratory, enteric viruses with inactivated vaccines.
3. FeLV vaccination for FeLV-negative cats has been associated with sarcomas. Vaccinate LOW on the LEFT hind.

88
Q

What is the earliest stage post-breeding that an experienced practitioner can reliably rectally palpate fremitus in the uterine artery of the gravid uterine horn in a pregnant cow?

A - 60-75 days
B - 45-60 days
C - 70-80 days
D - 120-150 days
E - 90 -120 days
A

d)
An experienced practitioner can reliably feel fremitus (vibration in uterine artery of the gravid uterine horn) between 120 and 150 days. Remember that remits in the uterine artery is a supportive of pregnancy but not conclusive evidence of pregnancy. An experienced practitioner can reliably feel the membrane slip of chorioallantoic membranes rectally at 30-35 days and can feel placentomes at around 70-75 days. Placentomes and membrane slip are cardinal signs of pregnancy.
One way to remember these is that the earliest occurs in reverse alphabetical order i.e.:
slip 30-35 days
placentomes 70-75 days
fremitus 90-120 days

89
Q

You serologically test 100 Siberian box turtles for galloping halitosis.
27 turtles test seropositive and 73 test seronegative.

However, molecular testing reveals 3/27 of the seropositive turtles are disease free and 10/73 of the seronegative turtles are diseased.

The entire fur-bearing turtle industry depends on your answer:
What is the Predictive Value Positive (PVP) of your serologic test?

A - 95%
B - 89%
C - 86%
D - 73%
E - 70%
A

b)
It is 89%. Remember- you are comparing two tests here. PVP means “Of the turtles my test says are positive (27), how many are truly positive?”

90
Q

A 1-year-old female llama is presented with hyperkeratosis around the nose and mouth.

What is the most likely diagnosis?

A - Munge
B - Dermatophilosis
C - Dorsal nasal alopecia
D - Wry face
E - Zinc-responsive dermatosis
A

a)
This is the clinical picture of munge (idiopathic nasal/perioral hyperkeratotic dermatosis). Some affected animals have lesions on the bridge of the nose and around the eyes and ears. Average age of one is 6 months to 2 years old.
Rx- address secondary bacterial infections (e.g., daily 10% povdone iodine scrubs and apply 7% tincture of iodine). If minimal response, include a topical glucocorticoid or intralesional triamcinolone acetone. If still no response, evaluate immune function.
Dorsal nasal alopecia is characterized by alopecia over the bridge of the nose with normal to variably scaly, hyper pigmented, and thickened skin. It is more common in dark-haired animal. It can be secondary to fly bites or rubbing the nose.
Animals with idiopathic hyperkeratosis (zinc-responsive dermatosis) present with thickened, nonpruritic papules with tightly adherent crusts in the less densely haired areas of the perineum, ventral abdomen, inguinal region, medial thighs, and axilla.
Signs can begin at any age.

91
Q

Which diseases are classically associated with a high anion gap (AG)?

A - Eosinophilic enteritis, hypothyroidism, gastrinoma
B - Prostatic adenocarcinoma, pleural effusion, lymphangiectasia
C - Diabetes mellitus, pancreatitis, proliferative enteritis
D - Hypercalcemia of malignancy, hypoadrenocorticism, displaced abomasum
E - Grain overload, ethylene glycol toxicity, renal failure

A

e)
Typically, expect an increased anion gap with: diabetic ketoacidosis, renal failure, ethylene glycol toxicity, lactic acidosis from grain overload or strenuous exercise. Anion gap is typically measured as positive cations (Na+, K+) minus negative anions (total CO2, Cl-). You can also calculate the anion gap using HCO3- in place of TCO2.
Anion gap= (sodium+potassium)-(TCO2+chloride).
Decreased anion gap is uncommon. Think hemodilution, hypoalbuminemia, hypercalcemia.

92
Q

A commercial pig farm experiences an explosive outbreak of respiratory disease with high mortality, primarily in young pigs under 6 months of age.

Affected pigs show severe respiratory distress, fever up to 107°F (41.5°C), anorexia, and reluctance to move. Some animals display open-mouth breathing with a blood-stained, frothy nasal and oral discharge.

On necropsy, the lungs are bilaterally dark and swollen and ooze bloody fluid from the cut surface.

Which one of the following choices is the most likely diagnosis?

A - Mycoplasma hyopneumoniae 
B - Fusobacterium necrophorum 
C - Actinobacillus pleuropneumoniae 
D - Haemophilus parahaemolyticus 
E - Swine influenza
A

c)
This is a classic scenario of an outbreak of pleuropneumonia, caused by Actinobacillus pleuropneumoniae. Primarily a growing pig disease, although adults can be affected as well, ans sows can abort. Mortality is high if untreated. Survivors may experience lowered growth rates and have a persistent cough.

93
Q

A poultry operation has been having problems with sick birds and many have died over the past few months.

A few showed torticollis before death. Some of the currently ill fowl have swollen joints, wattles, and footpads.

One of the broilers with swollen wattles is sacrificed. There is caseous, suppurative material inside the wattle and necrotic lung lesions.

Which one of the following agents is likely involved?

A - Fowlpox
B - Pasteurella multocida 
C - Marek's disease virus
D - Salmonella gallinarum 
E - Paramyxovirus
A

b)
Pasteurella multocida is the causative agent of fowl cholera. Older chickens are more vulnerable than are younger chickens. The description above is of chronic fowl cholera. In acute fowl cholera, there are usually no clinical signs before a large number of birds die. Classic lesions of Marek’s disease are enlarged sciatic nerves and lymphoid tumors. It is a highly contagious disease, but may be subclinical in many cases.

94
Q

Several 12 month-old sheep and goats are sick at a petting zoo that has cows, horses, pigs, bison and white-tailed deer. One of the deer is also affected. The rest of the animals appear well.

Affected sheep are febrile, listless and off-feed, with serous or mucopurulent nasal discharge and rectal temperatures ranging from 105-107.5°F / 40.5-42°C, [N=100.9103.8°F/38.339.9°C].

Physical exam shows swollen muzzles with edema and congestion of the lips, nose and face with small hemorrhages and ulcers on the mucous membranes. The ulcers appear where the teeth contact the swollen lips and tongue.

Two affected animals are lame.

What is the diagnosis?

shpnosesm.jpg

A - Bluetongue
B - Peste des petits ruminants (PPR)
C - Caprine arthritis encephalitis (CAE)
D - Contagious ecthyma
E - Ovine progressive pneumonia (OPP)
A

a)
Swollen sore muzzles with mucous membrane erosions, high fevers and lameness suggests infection with blue tongue virus. Bluetongue is almost exclusively seen in sheep, though white-tailed deer, pronhorn antelope and desert bighorn sheep in North America can be severely affected. Rare in cows. Reportable. Bluetongue is indistinguishable from foot and mouth disease (FMD), so that is a good second choice. But FMD is unlikely in the scenario above because FMD mainly affects pigs and cattle. Ovine progressive pneumonia (OPP) is a chronic disease of wasting and dyspnea in sheep and goats, caused by lentivirus (retrovirus family). App is most common in animals older than 4 years and rarely occurs in sheep and goats under 2 years of age.

95
Q

Which parasite can cause cutaneous larva migrans in people?

A - Habronema spp.
B - Trichuris spp.
C - Spirocerca sp.
D - Trichostrongylus spp.
E - Ancylostoma spp.
A

e)
Hookworws (Ancylostoma spp.) may cause cutaneous larva migrant in people. Note that roundworms (Toxocara spp., Toxasacaris spp., Baylisascaris spp.) are also zoonotic, causing visceral and ocular larva migrans in people. In dogs, whipworms, Trichuris spp. are associated with hypoadrenocorticism-like syndrome (hyponatremia, hyperkalemia, azotemia, metabolic acidosis). Whipworm infection has been suggested as one cause of cecocolic intussusception. Habronema spp. in horses can cause tumorlike stomach nodules and sometimes cutaneous lesions. Trichostrongylus spp. cause parasitic gastritis and enteritis in sheep, goats, and cattle.

96
Q

When a sow is giving birth, what is the shortest time interval between passage of piglets that indicates dystocia?

A - 4 hours
B - 1/2 hour
C - 2 hours
D - 1 hour
E - 15 minutes
A

d)
Dystocia in the sow is present when one hour or more has passed between piglets. Intervention should be swift to prevent death of piglets from anoxia. Dystocia is not common in swine but when it occurs, it is often due to uterine inertia, similar to other animal that produce multiple offspring. Other causes include fetal malposition, fetopelvic disproportion, obstruction of the pelvic canal, deviation of the uterus, and excitement in the dam. Medical therapy can be given with oxytocin and/or calcium only when an obstructive dystocia is not present.

97
Q

Most of a litter of European wild boar at a zoological park have died. The surviving piglets are in lateral recumbency with a frothy nasal discharge.

Necropsy of the piglets reveals pulmonary edema and copious fluid in the trachea and bronchi along with grayish- white necrotic foci on the myocardium.

Which one of the following diseases is the most likely diagnosis?

A - Encephalomyocarditis
B - Edema disease
C - Glasser's Disease
D - Porcine Respiratory and Reproductive Syndrome
E - Pseudorabies
A
a)
Encephalomyocarditis virus (EMCV) is caused by a cardio virus in the family picornaviridae. Confusingly, the virus is named for its effects on mice. Think of rodents and exotic zoo mammals with EMCV. Pig-to-pig contact, contamination of swine feed and water by rodents or ingestion of dead rodents may cause disease. See pulmonary deem and copious transudate in the respiratory tract, cousin cardiac failure. Zoo outbreaks of EMCV have included lions, African elephants, rhinos, hippos, sloth, llamas, antelope and nonhuman primates. An outbreak of lion deaths at a Florida zoo in the USA occurred after feeding them the carcass of an African elephant that had died of EMCV. 
Edema disease is a neurologic disease caused by a hemolytic Escherichia coli producing Shiga toxin e2 and F18 pili resulting in high mortality in recently-weaned pigs. Glasser's Disease, caused by Hemophilus paresis is usually an acute disease of 6 to 8 week-old pigs which causes fibrinous arthritis, polyserositis, and meningitis. Porcine Reproductive and Respiratory Syndrome (PRRS), is an arterivirus causing reproductive failure and post-weaning respiratory disease. Pseudorabies is a herpesvirus: infection causes CNS disease in neonates, respiratory disease in weaned pigs, and fever in all age.
98
Q

A 5 year old female spayed Cocker Spaniel is presented with a strange expression.

Her right ear and lip appear to droop.

There is ptosis O.D. (right eye) and the dog is drooling on the exam table.

What anatomic structure is damaged?

A - Facial nerve
B - Left side inner ear
C - Trigeminal nerve
D - Right side inner ear
E - Left side medulla, motor tract
A

a)
Think of facial nerve paralysis (CN 7) with a unilaterally droopy face. Remember the facial nerve is motor to the muscles of facial expression (explaining the right side drooped ear, lip and eyelid) and innervates the lacrimal and salivary glands. Loss of innervations can lead to a dry eye, and possibly to exposure keratitis if animal losses ability to close eyelid from damage to facial nerve innervation of the orbiculares oculus muscle. Idiopathic in 75% of canine cases (25% of cats). Can also see these signs with middle ear damage (from otitis media), from facial nerve trauma (ear surgery in dogs, or pressure from halter buckles in anesthetized horse), or neoplasia. Think more of a dropped jaw with trigeminal nerve neuropathy (CN 5).

99
Q

A petting zoo is experiencing a high mortality outbreak among young chickens, guinea fowl, pheasants and turkeys.

Affected birds huddle near heat sources. They are off feed and weak, with whitish feces pasted around their vents (diarrhea).

On necropsy, lesions include unabsorbed yolk sacs and gray nodules in the liver, spleen, lungs, heart, gizzard, and intestine. There are firm, cheesy cecal cores.

This presentation is highly suggestive of which diagnosis?

A - Campylobacter jejuni 
B - Pullorum disease
C - Avian spirochetosis
D - Histomoniasis
E - Coronaviral enteritis
A

b)
This is the clinical picture od so-called “pullout disease,” which usually causes very high mortality (potentially approaching 100%) in young birds. Until recently, pullout disease and fowl typhoid were thought to be causes by different bacteria. Genetic analysis has shown that in fact they are caused by two closely related biovars of the same bacteria, Salmonella enterica subsp. enteric serovar Gallinarum. Pullorum disease is caused by serovar Gallinarum biovar Pullorum (a.k.a Salmonella Pullorum). Fowl typhoid is caused by serovar Gallinarum biovar Gallinarum (a.k.a Salmonella Gallinarum).
Pullorum disease was once common in North America but has been eliminated from most commercial chicken stock. The disease may occur in other typer of avians (e.g., guinea fowl, quail, pheasants, sparrows, parrots, canaries and bullfinches).

100
Q

A 2-year-old male neutered Labrador retriever is presented with a history thunderstorm and fireworks phobia. The dogs clinical signs include pacing, tremors, house soiling, and vocalization.

Which one of the following choices is an appropriate medication regimen for a dog or cat exhibiting noise-related phobia/anxiety?

A - Alprazolam 1 hour before problematic event
B - Medications should not be given
C - Fentanyl patch as needed
D - Zolpidem on the day of a predicted fireworks show
E - Melatonin and coenzyme Q twice a day

A

a)
Administration of alprazolam 30-60 minutes to storms is one example of an appropriate medication regimen for thunderstorm phobias. Other anxiety-evoking events can include gunshots, owner departure, visits to the veterinarian, car rides, exposure to other dogs or strangers on a walk and visitors to the home. Anti-anxiety medications (e.g., alprazolam or diazepam) are useful for their rapid-onset of action. Administer 30-60 minutes prior to storms. Can use in combination with other psychotropic drugs such as selective serotonin reuptake inhibitors (SSRIs) or tricyclic antidepressants (TCAs) when managing concurrent behavioural issues such as separation anxiety. In addition to medication, behaviour modification should be implemented. Techniques include changing the dogs’ emotional response to there stimulus (counterconditioning), rewarding any reduction in the phobic behaviour and minimizing the pet’s exposure to panic-inducing stimuli.

101
Q

Which animal is considered to be as susceptible to acetaminophen toxicity as cats?

A - Dogs
B - Rabbits
C - Hamsters
D - Ferrets
E - Rats
A

d)
Ferrets are susceptible to acetaminophen toxicity. Like cats, ferrets metabolize acetaminophen slowly, because they are deficient in glucuronyl transferase. There are unsubstantiated web reports of pot bellied pigs reacting poorly to acetaminophen, but we have not yet seen peer-reviewed papers to verify this. There are recommended acetaminophen dosages for dogs, rabbits, rodents and guinea pigs, but high doses are toxic to dogs and other animals so don’t overdo it. In cats, toxicity can occur with 10-40 mg/kg. In one report, 3 of 4 dogs showed clinical signs of methemoglobineia at 200mg/kg, but toxicity can be seen at lower dosages with repeated exposures.

102
Q

One description of a typical heart sound is “lub-dub.”

What makes the first heart sound (S1) (i.e., the “lub”)?

A - Ventricular filling
B - Aortic and pulmonic valves
C - Mitral and semilunar valves
D - Atrial contraction
E - Atrioventricular valves
A

e)
The first heart sound (S1- the lub in lub-dub) is caused by closure of the atrioventricular valves (AV valves, mitral and tricuspid). The second sound (S2) is the closure of the aortic and pulmonic valves (semilunar valves). A third sound (S3) is the end of rapid ventricular filling and a fourth sound (S4) is atrial systole (atrial contraction). You can often hear all 4 sounds in horses, but typically hear only S1 and S2 in cattle and small animals.

103
Q

What is the most common cause of Horner’s syndrome in dogs?

A - Idiopathic
B - Brachial plexus avulsion
C - Neck bite wounds
D - Otitis media
E - Retrobulbar neoplasia
A

a)
Horner’s Syndrome is idiopathic in 50% of dogs and 45% of cats.
It is caused by a lack of sympathetic innervation to the eye.
See four ocular signs with Horner’s: 1. Miosis (constricted pupil) 2. Protrusion 3rd eyelid (nictitans) 3. Enophthalmos (sunken eye) 4. Ptosis (drooped eyelid, +/- anisocoria.
Look for classic increased anisocoria in the dark. Other diagnostic tests depend on where suspect primary disease. If you can get a primary diagnosis, (e.g. Guttural pouch mycosis in horses, brachial plexus avulsion, otitis media, etc.) must treat that.

104
Q

Pyrrolizidine alkaloid toxicity is caused by chronic ingestion of which one of the following plants?

A - Astragalus spp. (locoweed)
B - Lupinus spp. (lupine)
C - Nerium spp. (oleander)
D - Senecio spp. (ragwort)
E - Persea spp. (avocado)
A

d)
Common plants containing pyrrolizidine alkaloids (PA) are: Senecio vulgaris, S. jacobea, Amsinckia intermedius, Heliotropium europaeum, Crotolaria spectabilis. Although generally not palatable, livestock will eat there plants when baled in hay or on pasture when forage is scarce. Chronic ingestion allows accumulation of toxic levels of PA, resulting in hepatic fibrosis. Poisoning is most common in horses and cattle. Sheep and goats ar more resistant to toxic effects. Hepatic pathology with associated clinical signs is the most common manifestation of pyrrolizidine alkalosis in domestic animal species. Clinical signs may not be seen for several weeks of months after initial exposure. Consumption of the offending plant may even have ceased months earlier. Clinical signs include loss of body condition, anorexia, dullness, and constipation of diarrhea. Photosensitization and hepatoencephalopathy can also occur.
Persea spp. (avocado leaves) contain person. Signs of toxicity are: non infectious mastitis, abrupt cessation of milkflow, heart failure. Nerium spp. (oleander) contain cardiac glycosides. Signs of toxicity are: sudden death, weakness, diarrhea, cardiac arrhythmias. Lupinus spp. (lupine) contain alkaloids. Signs of toxicity are: birth defects (ingestion at 40-70 days in cattle), abortion, tremors, incoordination, head pressings, seizures. Astragalus and Oxytropis (locoweed) contain alkaloids (swainsonine). Signs of toxicity are: excitability, incoordination, difficulty eating, exaggerated mouth movements, depression.

105
Q

Which one of the following sets of conditions predispose a cow to metritis?

A - Dystocia, overfeeding in dry period, Ca-P imbalance in feed
B - Agalactia, milk fever, bovine vibriosis
C - Strep. agalactiae mastitis, retained placenta, laminitis
D - Milk fever, malnutrition, excess dietary zinc
E - Contaminated calving environment, abortion, hypomagnesemia

A

a)
Cows are predisposed to post-parturient bovine metritis by many things, including: dystocia, overfeeding in dry period, Ca-P imbalance in feed.
Cows are also predispose to metritis by: retained placenta, contaminated calving environnement, abortion, malnutrition.
In cattle, the causative bacterial organisms isolated most often are Trueperella progenies alone or with Fusobacterium necrophorum or other gram-negative anaerobes. Specific diseases associated with bovine metritis or endometritis include brucellosis, leptospirosis, trichomoniaisi and bovine campylobacter.

106
Q

A six-year-old female spayed golden retriever is presented for acute onset of flaccid jaw paralysis.

The dog is unable to close her mouth, has difficulty eating and is drooling. There is no history of trauma.

What is the most likely diagnosis?

A - Masticatory myositis
B - Facial nerve paralysis
C - Trigeminal neuritis
D - Myasthenia gravis
E - Meningoencephalitis
A

c)
Idiopathic trigeminal neuritis, involves cranial nerve 5, (CN5) and is characterized by acute onset of flaccid jaw paralysis. Affected animals cannot close their mouth and have difficulty eating and drinking. In some cases unilateral or bilateral Horner’s syndrome, facial paresis, and decreased facial sensation are present. Most common in dogs, rare in cats. Cause is unknown.
Idiopathic facial nerve paralysis, cranial nerve 7 (CN7) results in the inability to move the eyelid, lip or ear and dryness of the eyes and mouth. Masticatory myositis, an immune-mediated disease of dogs, is characterized by pain on opening the mouth and swelling of the muscles of mastication (acute) or atrophy of the temporals and massage muscles with the inability to open the mouth due to fibrosis (chronic). Myasthenia Gravis (MG), a neuromuscular disease, typically presents with episodic or exercise-induced weakness due to impaired transmission of acetylcholine at the neuromuscular junction of skeletal muscles. Meningoencephalitis, inflammation of the brain and meninges, should always be on the differential list. Lesions may be anywhere in the CNS and may present with any neurological signs.

107
Q

An aged intact female potbellied pig presents for evaluation of abdominal distension.

A large mass contiguous with the uterine wall is discovered with ultrasound exam of the abdomen.

Which of the following choices is the most likely cause of this finding?

A - Uterine leiomyoma
B - Endometritis
C - Adenocarcinoma of the cervix
D - Lymphosarcoma
E - Endometrial hyperplasia
A

a)
The most likely cause of a large uterine mass intact female potbellied pig is a uterine leiomyoma. These tumors do not typically metastasize so a cure may be achieved by surgical removal of the uterus.

108
Q

During a routine immunization visit for a two-year-old neutered male Newfoundland dog, a systolic ejection-type (crescendo-decrescendo) murmur is detected.

It is audible loudest on the left side of the chest between the 2nd and 5th intercostal (IC) space and at the thoracic inlet lateral to the trachea.

Which condition is highest on a differential diagnosis list?

A - Pulmonic stenosis
B - Mitral dysplasia
C - Tricuspid dysplasia
D - Aortic stenosis
E - Patent ductus arteriosus (PDA)
A

d)
Aortic stenosis (also called sub-aortic stenosis), is a systolic, ejection-type (crescendo-decrescendo) heart murmur which may be heard most loudly on the left chest between the 2nd and 5th intercostal space (IC) or at the thoracic inlet (lateral to trachea). Inherited in Newfoulands. Predilection in many big breeds- german shepherd, golden retriever, boxer, rottweiler.
Mitral dysplasia and other mitral valve problems are heard further back on left at 5th-6th IC. More common in cats.
With pulmonic stenosis see right ventricular hypertrophy, because pulmonic valves blocks outflow from R ventricule (mostly dogs).
Tricuspid dysplasia is heard further back on right at 5th-6th IC> Uncommon.
Expect a continous murmur with patent ductus arterioles (PDA). Vast majority detected at first vaccination visit.

109
Q

Which drug is PERMITTED for use in food animals?

A - Diethylstilbesterol
B - Chloramphenicol
C - Dexamethasone
D - Estradiol cypionate
E - Furazolidone
A

c)
Dexamethasone is allowed in food animals, but may cause abortion in pregnant animals. Typically, use dexamethasone in cattle to induce parturition (20-30 mg, IM, given within 2 wk of normal term).
Diethylstillbesterol (DES) is banned for use in food producing animals and should never be used.
Chloramphenicol has been associated with bone marrow suppression/aplastic anemia in exposed humans, and is contraindicated in food-animals.
According to a 2006 report from the food and drug administration (FDA) the se of estradiol cypionate (ECP) in animals is illegal. ECP has been used as an estrogenic hormone fro reproductive therapy in food animals, but even extra-label, this is not allowed.
Furazolidone a nitrofuran, is not allowed in food animals.

110
Q

Which choice is associated with osteosarcoma and hypertrophic osteopathy?

A - Spirocerca lupi 
B - Coccidioidomycosis
C - Habronema spp.
D - Blastomycosis
E - Ollulanus tricuspis
A

a)
A disease of dogs in the Southern US and tropical climates, Spirocerca lupine (esophageal worms) make reactiove granulomas of variable size in the esophageal, gastric or aortic walls. Large granulomas may become neoplastic (osteosarcoma, fibrosarcoma). Some dogs develop spondylitis or enlargement of the extremities characteristic of hypertrophic osteopathy. Typically asymptomatic, but large granulomas can cause esophageal obstruction. Spirocercosis may also lead to aneurysm in the thoracic aorta or an ossifying spondylitis of the posterior thoracic vertebrae. Habronema app. in horses can cause tumorlike stomach nodules and sometimes cutaneous lesions. Blastomycosis most common in dogs, cats and humans is characterized by pyogranulomatous lesions in various tissues. Placental infection in horses with Coccidioides immitis have been described, leading to abortion and osteoporosismuelitis.

111
Q

A seven-week-old male Yorkshire terrier is presented with a two-week history of on and off vomiting and diarrhea that began around the time he was weaned.

The owners relate that he seems to “drink and pee a lot.”

They report pacing, disorientation, weakness, and “stumbling around.”

Physical exam is unremarkable, but only one testicle has descended.

As the puppy explores the room he appears ataxic, stumbles a few times, and bumps his head into the wall.

What is the clinical diagnosis?

A - Congenital hiatal hernia
B - Portosystemic vascular shunt
C - Canine distemper
D - Diabetes insipidus
E - Lead poisoning-encephalopathy
A

b)
Portosystemic vascular shunt.
Signs of hepatic encephalopathy (ataxia, disorientation, vomiting, diarrhea) beginning after weaning combined with polyuria/polydipsia (“drink and pee a lot”) in a cryptorchid male Yorkshire terrier says congenital portosystemic vascular anomaly. (PSVA, also referred to as a portosystemic shunt).
PSVA are seen most in pure-breeds. Think small- Mlatese, yorkshire terrier, miniature schnauzers (but can also see in old english sheepdogs, irish wolfhounds). Usually in young animals, especially after weaning. Cryptochidism is common.
Lead poisoning can cause vomiting, diarrhea and CNS signs (blind, hyperactive seizures). Can see PU/PD in older animals but Hx here puts PSVA first on DDX.

112
Q

The major active ingredient in most IV euthanasia solutions is:

A - Phenobarbital
B - Phenytoin
C - Pentobarbital
D - Thiopental
E - Potassium chloride
A

c)
Pentobarbital is one of the most commonly used drugs in IV euthanasia solutions. The AVMA report states IV injection of a barbituriques acid derivative is the preferred method for euthanasia of dogs, cats, other small animals and horses. Potassium chloride can be used in conjunction with general anesthesia. Phenytoin and lidocaine are often added to pentobarbital-containing euthanasia solutions to increase cardiac depressant effects. Thipental is an ultra-short acting thiobarbiturate used for anesthesia inductions and for short procedures. It is currently unavailable in the Us and Canada.

113
Q

A six-year-old hunter-jumper horse is presented with a three-month history of poor performance and intermittent shifting hindleg lameness.

There is reluctance to bring the hind limbs forward at a trot and he occasionally drags his hind toes.

On physical exam, there is poor muscling of the gluteal muscles and some asymmetry to the croup (rump). The horse shows pain and a reluctance to ventroflex the back when midline pressure is applied.

Which one of the following is a top differential diagnosis?

A - Suspensory desmitis
B - Coxofemoral luxation
C - Sacroiliac disease
D - Overlapping vertebral spinous processes
E - Cauda equina (polyneuritis equi)
A

c)
Think of sacroiliac (SI) disease in an athletic jumping horse with:
-intermittent hindlimb lameness localized in the croup (rump)
-evidence of back pain
-swelling over the tuber sacrale (“hunter’s bumps”)
Horses with hunter’s bumps can be sound if the injury has completely healed, but lameness can be observed if the injury is recent or has never resolved. It is important to rule out other causes of hind end lameness or back pain/strain. Ultrasound per rectum or local anesthesia infiltration of the SI region can help confirm the diagnosis. However, most often treatment is administered instead (i.e., inject area with steroids and analgesics) and the response to therapy monitored.
Coxofemoral luxation is rare in horses due to a deep acetabulum and the presence of the accessory ligament of the hip, unique to horses.
Overlapping vertebral spinous processes (a.k.a. “kissing spines”) is a problem of thoracic and lumbar vertebra under the saddle area, seen in short-backed eventing horses (hunter/jumpers, dressage).
Cauda equina (polyneuritis equi) can cause gluteal muscle atrophy, but also causes tail paralysis, urinary and fecal incontinence, perineal analgesia or parenthesis, and mild hind limb ataxia.

114
Q

Which two diseases would be on a differential diagnosis list for equine urinary incontinence?

A - Bracken fern toxicity, equine encephalomyelitis
B - Equine degenerative myelopathy, botulism
C - Nigropallidal encephalomalacia, locoweed poisoning
D - Cauda equina neuritis, herpesvirus myelo
E - Tetanus, West Nile virus encephalopathy

A

d)
Cauda equina neuritis and herpesvirus myelencephalopathy.
Horses with caudal equine neuritis (also called polyneuritis equine) have a progressive symmetric LMN paresis of the tail, bladder, rectum, anal sphincter. Look for urinary incontinence, fecal retention, and a weak or paralyzed tail. May see hind limb paresis if lumbosacral cord is affected. Cranail nerves can also be affected, but typically cranial involvement is asymmetric. May see temporal or master atrophy (cranial nerve 5), facial paralysis, and exposure keratitis (cranial nerve 7), head tilt, or other CNS signs. Cause i unknown, may be an autoimmune process. Grave prognosis. Eventually euthanized.
Herpesvirus myeloencephalopahty (EHV-1) may also present with urinary incontinence, but this is a relatively uncommon manifestation of equine rhinopneumonitis. You would expect to hear a history of the more common EHV signs in other horses from the same farm, like respiratory disease (“snots”) in foals and abortions in mares.
Bracken fern toxicity (Pteridium aquiline) causes thiamine deficiency in monogastrics (like horses) and bone marrow depletion, aplastic anemia and bladder tumours in ruminants. In horses, look for signs of anorexia, weight loss, incoordination, and a crouching stance with feet placed wide apart. Horses may have trembling muscles when forced to move.

115
Q

Which of the following nerves are targeted with a paravertebral block used to perform a standing laparotomy in a cow?

A - L1-3, and S1-5
B - T13, L1, and L2
C - L2, L3, S1, and S2
D - L1, L2, L3
E - L1, L2, and L4
A

b)
The spinal nerves, T13, L1, and L2 must be blocked to completely desensitize the flank of a cow. The paravertebral (PV) nerve block targets these nerves. It can be performed via two techniques- the proximal or distal PV block. The proximal block places local anesthesia in the space just caudal to the transverse processes of the vertebrae- T13, L1 and L2. The distal block is placed at the ends of the transverse processes of the vertebra- L1, L2 and L4 as there nerves gradually course caudally after they exit the spinal foramen. Proper placement of the anesthetic results in warming of the skin from vasodilation, anesthesia of the skin and body wall, and a curvature of the spine in some cows. The latter is caused by relaxation of the epaxial musculature on the affected side; the spine curves in a convex manner.

116
Q

A four-day-old foal is presented with clinical signs of depression, icterus, head pressing, and disorientation.

Serum biochemistry reveals increased levels of gamma-glutamyltransferase, alkaline phosphatase, and unconjugated bilirubin.

On further questioning, the owner mentions that he gave the foal an oral nutritional supplement when it was a day old but was not sure what was in it.

‘What could this supplement have contained to cause these clinical signs?

A - Vitamin E
B - Copper
C - Iron
D - Vitamin A
E - Selenium
A

c)
Neonatal foals given oral iron supplements at birth can develop toxic injury to their liver and subsequent hepatic encephalopathy. Foals are born with a high serum iron level and absorb iron more readily than adult horses.

117
Q

A canine coagulation profile indicates increased values for APTT, PT, TT and FDPs.

Thrombocytes=62,513 per microliter (normally greater than 211,000 per microliter)

What is the typical prognosis for a dog with the condition suggested by this pattern?

A - Good
B - Fair
C - Grave
D - Excellent
E - Need more information
A

c)
Grave
A lab pattern of low platelets and across the board increases in bleeding time, APTT, PT, TT and FDP tests suggests disseminated intravascular coagulation (DIC). Remember that DIC also stands for “ death is coming”. Prognosis is typically grave and mortality is high even with treatment. DIC is not a disease in its own right. It is a complex hemostatic defect characterized by enhanced coagulation and fibrinolysis, secondary to other diseases. Fibrinolysis and depletion of clotting factors leads to hemorrhage. Many diseases, all of them bad, can precipitate DIC. Remember your H diseases associated with DIC: heartworm, heart failure, hemolytic anemia, hemangiosarcoma, hemorrhagic gastroenteritis, hepatic disease, esp. hepatic lipidosis in cats.
Gastic dilatation-volvulus (GDV), mammary gland carcinoma and pancreatitis can also lead to DIC.

118
Q

Which one of the following is the most common cause of dystocia in cattle?

A - Uterine inertia
B - Fetal monsters
C - Maternal birth canal abnormalities
D - Abnormal fetal presentation, position, and/or posture
E - Fetopelvic disproportion
A

e)
Fetopelvic disproportion, where the fetus is too large to pass through the maternal pelvis canal, is the most common cause of dystocia in cattle. It is a significant cause of calf losses, especially in first-calf heifers. Fetopelvic disproportion may occur in small heifers with normal sized calves or in cows with large fetuses. Dystocia occurs on average in 10-15% of first-calf heifers and in 3-5% of mature cattle. The incidence of dystocia in heifers increases as the age and size of replacement heifers decreases. The size of the sire and other factors also contribute to the incidence of dystocia. Close monitoring of heifers and early intervention are necessary to prevent calf loss.

119
Q

An adult horse with clinical signs of voluminous gastric reflux, depression, colic, and fever is most likely suffering from which condition?

A - Ulcerative duodenitis
B - Proliferative enteropathy
C - Proximal enteritis
D - Right dorsal colitis
E - Cantharidin toxicity
A

c)
Proximal enteritis, or duodenitis-proximal jejunitis, is a clinical syndrome characterized by large volumes of gastric reflex resulting from excessive fluid and electrolyte secretion into the small intestine and small intestinal inflammation and edema.
Laminitis is an important potential sequela.
The cause is unknown but several bacteria and toxins including Clostridium difficile, Clostridium perfringens, Salmonella and fumonosin B1 mycotoxins have been implicated. Ulcerative duodenitis is a disorder of foals resulting in fever, colic, diarrhea, and delayed gastric emptying. Lawsonia intracellular causes proliferative enteropathy in foals and weanlings, a disease characterized by hypoproteinemia, diarrhea, chronic ill thrift, and ventral edema. Right dorsal colitis is typically a result of non-steroidal anti-inflammatory drug toxicity and results in hypoproteinemia and colic. Cantharidin toxicity (blister beetles, Epicauta spp. ) causes a wide range of clinical signs predominated by profuse diarrhea, strangurie and pollakuria, and colic.

120
Q

A seven-year old female spayed Border Collie is presented with two very goopy, gunk-covered eyes.

A Schirmer tear test reveals less than 10 mm/minute of wetting [N= 15mm or more/ minute].

The owner reports that the dog has been on “some kind of medicine” for the last ten days, but it is not his dog, and he doesn’t know what the medicine is.

Keratoconjunctivitis sicca (KCS) secondary to the drug is suspected.

Which one of the following drugs may be causing the KCS?

A - Amitraz
B - Trimethoprim-sulfa
C - Griseofulvin
D - Itraconazole
E - Prednisolone
A

b)
Systemic sulphonamides like trimethoprim-sulfa have been associated with keratoconjunctivitis sick (KCS), sometimes irreversibly. Another drug-associated cause of transient KSC is the combination of recent general anesthesia and atropine. Other causes of KCS include: distemper, immunologic (think ATOPY), breed (PUGS, Yorkies), and trauma (proposed eyeballs).

121
Q

A bovine diet that is low in thiamine or high in sulfur can cause ____________.

A - Polioencephalomalacia
B - Pregnancy toxemia
C - Parturient paresis
D - Downer cows
E - Pseudorabies
A

a)
The two most common causes of polioencephalomalacia (PEM) are:
-Low thiamine (due to thiamine activity from plants like bracken fern of low thiamine diets)
-high sulfur in the diet (which can come from a high molasses-urea diet, corn or sugar cane byproducts, water, or other plants, including alfalfa, Canada thistle, kochi, and lambsquarter

122
Q

Feline leukemia (FeLV) status has been shown to affect many aspects of disease progression and treatment of lymphoma.

FeLV seropositive cats with lymphoma are similar to FeLV seronegative cats with lymphoma in which one of the following aspects?

A - Frequency of spinal lymphoma
B - Initial treatment response
C - Survival time
D - Prevalence of gastrointestinal lymphoma
E - Prevalence of mediastinal lymphoma
A

b)
Feline leukemia virus seropositive and seronegative cats share similar response rates to initial treatment with chemotherapy for lymphoma. However, survival times are shorter for cats with FeLV compared with FeLV negative cats. Most US cats with mediastinal, multi centric, or spinal from of lymphoma are FeLV-positive.

123
Q

A two-year-old budgerigar (parakeet) is presented with a history of limping on his left leg, which has gotten progressively worse over the last three weeks.

He is eating and drinking normally, but has fallen off his perch twice in the last two days.

Weakness in the left leg compared to the right one is observed.

He retracts his left more slowly and more weakly when his toe is pinched.

What condition is at the top of the differential diagnosis list?

A - Pacheco's disease
B - Polyoma virus infection
C - Renal tumor
D - Trauma - broken leg
E - Hypovitaminosis A
A

c)
Think renal tutor with progressive unilateral lameness in budgerigars (and poor prognosis). The sciatic nerve passes through the renal parenchyma in birds, and budgerigars are prone to neoplasia in general.
Avian polyomavirus is a young bird disease characterized by lethargy, crop stasi, death, and sometimes subs hemorrhages after injections.
Pacheco’s disease- think “sudden death” in younger birds.
Hypovitaminosis A- Think sinusitis, “bu-eyes” (exophthalmia) sublingual abscesses, white plaques. Broken bone possible, but would expect acute onset, not progressive.

124
Q

Which nerve block would most specifically relieve lameness resulting from fracture of the navicular bone?

A - Low four-point
B - Abaxial sesamoidean
C - Palmar digital
D - Median and ulnar
E - Tibial and peroneal
A

c)
The palmar digital nerve block (also called a PD or heel block) would anesthetize at least the palmar third of the foot, including the navicular bone.
Historically, the PD block was considered to anesthetize the palmar/plantar 1/3 of the foot and sole. However, it has been shown that it desensitizes 70-80% of the foot in mort horses. All of the other nerve blocks can desensitize the navicular bone too, but would not be as specific as a palmar digital nerve block. The abaxial sesamoidean nerve block (ASNB, also called a pastern or foot block) would anesthetize the entire foot and much of the pastern. Sometimes an ASNB is needed to completely eliminate or diagnose navicular lameness, but it is less specific the the PD block. Allow four-point nerve block (also called low palmar or solar block) would desensitize the fetlock and areas distal to it. The median and ulnar nerve block would anesthetize the carpus and areas distal to it. The tibial and perineal nerve block would anesthetize the tarsus and areas distal to it.

125
Q

Which lesions are most commonly associated with Marek’s disease?

A - Edematous facial swelling with sinusitis
B - Hemorrhagic skin lesions
C - Nerve enlargement
D - Wrinkled eggs
E - Blood in the trachea
A

c)
Think of nerve enlargement with Marek’s disease. Also can see distortion of the pupil and enlargement of feather follicles (“skin leukosis”=condemnation of carcass). May see one leg forward, one leg back, a transient paralysis. Think infectious laryngotracheitis (ILT) if there is blood occluding trachea on necropsy. Remember wrinkled eggs go with infectious bronchitis. Think infectious coryza with sinusitis, swelling under eyes.

126
Q

What is the proper name for the swine kidney worm?

A - Capillaria plica 
B - Stephanurus dentatus 
C - Dioctophyma renale 
D - Gnathostoma spinigerum 
E - Trichinella spiralis
A

b)
Stephanurus dentatus is the swine kidney worm. Found worldwide, particularly in tropical and subtropical areas. Seen in the US as a parasite of pigs raised outside in the southeastern and south-central states.

127
Q

Which one of the following choices is a competitive antagonist for aldosterone that is sometimes used in treatment of congestive heart failure?

A - Hydrochlorothiazide
B - Furosemide
C - Mannitol
D - Acetazolamide
E - Spironolactone
A

e)
Potassium-spatring diuretics (i.e.. spironolactone) are aldosterone antagonists sometimes uses with furosemide as part to the treatment plan for congestive heart failure (CHF).
Aldosterone, the mineralocorticoid produced in the adrenal cortex, acts on the distal convoluted tubules (and collecting ducts) of the nephron to retain Na+ and water, secret K+ and increase blood pressure. With CHF, aldosterone levels are increased due to activation of the renin-angiotensin-aldosterone system. In contrast, see decreased aldosterone secretion (and decreased blood pressure) with hypoadrenocorticism.

128
Q

What is the average length of estrus in the cat?

A - 2-3 days
B - 4-5 days
C - 9-10 days
D - 6-7 days
E - 21 days
A

d)
Estrus in cats generally lasts 6–7 days (range 1-10 days). Cats are seasonally polyestrous and induced ovulators. The length of feline estrus is affected by wether a male is present. If a male is present, estrus typically lasts 1-4 days. Without a male, estrus lasts 7-10 days and recurs in 2-3 weeks. Unlike dogs, the feline estrus cycle is controlled by day length. In North America, cats go through an anestrus period in December and January when day length is less than 12 hours.

129
Q

A one-year-old intact male domestic short hair cat is presented with dyspnea after vehicular trauma.

On physical examination, the cat is open-mouth breathing with muffled heart sounds and borborygmi heard on pulmonary auscultation.

Which one of the following choices is the most likely diagnosis?

A - Cardiac tamponade
B - Diaphragmatic hernia
C - Posterior lung lobe torsion
D - Flail chest
E - Pneumothorax
A

b)

Diaphragmatic hernia is most likely to cause dyspnea with gastrointestinal sounds auscultable in the chests.

130
Q

Atresia coli in the calf is associated with which one of the following choices?

A - Freemartins
B - Rectal palpation in 1st 42 days of pregnancy
C - History of ketosis/hepatic lipidosis in dam
D - Oral doxycycline
E - Dry matter intake over 20% during dam’s peak lactation

A

b)
Vascular damage secondary to amniotic vesicle palpation in the first 6 weeks of embryonic development can lead to intestinal ischémie and atresia in calves. Inherited atresias of the intestine are relatively common in large animals. Only 30% of calves with atresia coli survive to adulthood. Surgical correction is not recommended in Holstein calves because atresia coli is probably heritable in this breed.
Congenital atresia ani, (imperforate anus, seen most in cattle, sheep, pigs) occurs when the dorsal membrane separating the rectum and anus fails to rupture. Clinical signs at birth include tenesmus, abdominal pain and distention, retention of faces, absence of an anal opening.

131
Q

Which one of the following diseases is characterized by schistocytes on a complete blood count?

A - Congenital hepatic arteriovenous fistula
B - Disseminated intravascular coagulation
C - Cyanocobalamin deficiency
D - Methemoglobinemia
E - Acetaminophen toxicity

A

b)
Disseminated intravascular coagulation. Schistocytes are produced by the mechanical fragmentation of erythrocytes on intravascular fibrin strands. They are most commonly associated with conditions that affect blood flow or clotting like disseminated intravascular coagulation (DIC), heart worm, hemangiosarcoma, and glomerulonephritis. Cobalamin deficiency causes a microcytic anemia (pernicious anemia). Systemic lupus erythematous may cause anemia, thrombocytopenia, and leukopenia or keukocytosis. Think of methemoglobinemia, Heinz bodies, and hepatoxocity with acetaminophen toxicity in cats.

132
Q

What is the neuroanatomic cause of extensor rigidity in the forelimbs of a cat with Schiff-Sherrington syndrome?

A - Interruption of inhibitory neuron input from lumbar spinal cord
B - Brachial plexus trauma
C - Damage to thoracic spinocerebellar tracts in superficial white matter
D - Polyradiculoneuritis of peripheral nerve sheaths at cervical intumescence
E - Subdural white matter compression at C6-T3

A

a)
Interruption of inhibitory neuron inpu from lumbar spinal cord. with severe trauma between T2-T13, inhibitory pathways (esp. around L2-L4) are interrupted and cervical intumescence neutrons (C6-T2) are “released”, causing extensor hypertonia in the forelimbs.
Think of schiff-sherrington syndrome when you see a combination of forelimb extensor rigidity and hind limb flaccid paralysis, in an animal that has just had major spinal trauma, like being hit by a car.

133
Q

Which one of the following vaccines is considered a core vaccine for South American camelids?

A - Rabies
B - Tetanus
C - Listeria
D - Equine herpes virus I
E - Bovine viral diarrhea virus
A

b)
Tetanus and Clostridium perfringens type C and T toxoids are the core vaccines recommended in South American Camelids. Rabies vaccination is indicated in rabies-endemic areas. Use of the vaccine for EHV-1 in camelids (when exposed to exotic equids) has caused blindness and should only be used when necessary. Vaccination for BVDV may provide an antibody response and protection when exposed. No vaccines are licensed for use in camelids.

134
Q

Which nerve block predicts how a horse with caudal heel pain (also called navicular disease) may respond to digital neurectomy?

A - High 4-point block
B - Local infiltration of suspensory ligament
C - Abaxial block
D - Pastern joint block
E - Palmar digital block
A

e)
A palmar digital block (also called the heel block) will anesthetize the bulbs of the heel. This mimics the effect of a palmar digital neurectomy, the last-ditch treatment to lengthen the athletic life of a horse with caudal heel pain (navicular disease). Remember that neurectomy will eliminate pain, but not disease- you do it when there has been no improvement from medical Rx after 6-12 weeks.

135
Q

Which of the following pairs of tissues can both cause increased serum alanine aminotransferase (ALT) levels in dogs and cats?

A - Pancreas and intestine
B - Kidneys and pancreas
C - Liver and muscle
D - Spleen and adrenals
E - Heart and kidneys
A

c)
Damage to liver and muscle cells of dogs and cats causes increased serum levels of alanine aminotransferase (ALT). ALT is considered a “leakage” enzyme. High levels are normally found in the cytoplasm of healthy cells. When hepatic or muscle cells are damaged, ALT leaks into adjacent tissue where it is picked up by the venous circulation. Other animals such as horses, ruminants, pigs, and birds do not have high levels of ALT inside cells. Aspartate aminotransferase (AST) is the predominant leakage enzyme in these animals. The magnitude of the elvation of leakage enzymes can be deceiving -are a few cells leaking a lot or many cells are leaking a small amount? Sever damage to a healthy liver may result in very high levels, while low levels may be seen when significant atrophy or fibrosis of the liver is present and few cells are left. Prognosis of the former may be good, for the latter it is certainly grim. Differentiation between muscle and liver as the source of increases in ALT/AST is determined by evaluation of other muscle (creatine phosphokinase and AST) and liver enzymes (AST, sorbitol dehydrogenase (SDH), gamma glutanyl transferase, alkaline phosphatase) the animal and the clinical evaluation. Leakage enzymes do not provide an estimate of actual liver function. This is evaluated by measurement of substances dependent on the efficacy of liver function such as bile acids, blood urea nitrogen, albumin, and glucose.

136
Q

When a foal is being treated with erythromycin (for Rhodococcus equi, for example), the mare is at risk of developing enterocolitis due to which one of the following organisms?

A - Escherechia coli 
B - Clostridium novyi 
C - Lawsonia intracellularis 
D - Rhodococcus equi 
E - Clostridium difficile
A

e)
A history of recent antimicrobial therapy is common in cases pf Clostridium difficile associated diarrhea. Adult horses exposed to erythromycin are particularly at risk for C. difficile enterocolitis. Clostridium novae is the cause of infectious necrotic hepatitis, which is primarily seen in sheep but can also be seen in cattle , hogs, and horses. Lawsonia intracellular is the cause of proliferative enteropathy, resulting in diarrhea and hypopreteinemia in foals and swine.. Rhodococcus equine is a notable cause of pneumonia in older foals characterized by pulmonary abscessation as well as some extra pulmonary manifestations. Escherechia coli can be cause of septicaemia and diarrhea in foals and calves.

137
Q

Which one of the following choices is a major clinical complication of equine pituitary pars intermedia dysfunction?

A - Dyshormonogenetic goiter
B - Circling, head-pressing
C - Diabetes insipidus
D - Colic
E - Laminitis
A

e)
The major clinical complication of pituitary pars intermedia dysfunction (PPID), also known as equine Cushing’s disease) is laminitis. Laminitis affects more than 50% of the horses with PPID. Additional clinical signs include hirsutism, polyuria/poludipsia, muscle wasting, regional adiposity, decreased mentation, and immune suppression.

138
Q

A two-year-old Angus cow is presented with weakness of the hind limbs and a staggering gait.

On physical exam, the heart rate and respiratory rate are slow and irregular, and the pupils are dilated.

The cow’s breath and urine smell like the odor of mouse urine.

Suddenly the cow’s pulse becomes rapid and thready, she collapses, and dies of apparent respiratory failure.

Which one of the following plants is most likely to have caused this spectrum of clinical signs?

A - Pinus ponderosa (Western yellow pine)
B - Veratrum spp (False hellebore, Skunk cabbage)
C - Centaurea spp (Knapweed, Yellow star thistle)
D - Conium maculatum (Poison hemlock)
E - Tetradymia spp (Horsebrush)

A

d)
Poisoning with Conium maculate (Poison hemlock) is responsible for the death of this cow. At least 8 toxic piperidine alkaloids have been isolated from poison hemlock. Coniine is found in seeds and the mature plants; g-coniceine is found in young growing plants. Poison hemlock is toxic to all livestock and humans. Signs of toxicity develop 1-2 hours after ingestion and are usually fatal. Signs include nervousness, trembling, weakness especially of hind limbs, weak pulse, irregular heart rate, recumbency, coma and death. A mousy door exuding from the urine and breath is pathognomonic. Ingestion of poison hemlock during gestation causes arthrogryposes and other congenital defects in cattle, goats, and pigs.

139
Q

You are presented a healthy ten-year-old maiden mare for breeding evaluation in January in North America.

On rectal palpation, both ovaries have multiple, small (less than 15 mm) follicles and no corpus lutea.

The uterus is flaccid with no edema.

Which one of the following choices is the most likely explanation of the findings in this mare?

A - Seasonal anestrus
B - The mare is in diestrus
C - Estrus has just passed
D - Early pregnancy
E - The mare is too old and now is infertile
A

a)
Seasonal anestrus.
Males are seasonal breeders and enter anestrus in the short-day months of late fall and winter. In anestrus the ovaries are inactive with small follicles and no corpus luted. During estrus, the uterus softens and multiple follicles form on the ovaries. During pregnancy the uterus has good tone.

140
Q

What is the purpose of placing a permanent rumen fistula in some cattle?

A - This procedure is not done on a permanent basis in cows
B - Allow for direct administration of medications into rumen long-term
C - Manage vagal indigestion in the fistulated animal
D - Create transfaunation donors
E - Treat traumatic reticuloperitonitis in the fistulated animal

A

d)
The rumen is permanently fistulated in the left paralumbar fossa to allow cows to be donors for transfaunation of rumen contents to other cows. Transfaunation is used to threat other cows with conditions like luminal acidosis, vagal indigestion and peritonitis. Fresh ruminal fluid contains microbes (bacteria and protozoa), volatile fatty acids (VFAs), microbial proteins, vitamins, minerals, and other buffers. Ruminal fluid can also be collected via siphon from a stomach tube or from animal at the slaughter house. Ruminal fistulas are also used to study ruminant physiology.

141
Q

Failure to control mild endemic respiratory disease in swine caused by Mycoplasma hyopneumoniae predisposes pigs to complications.

Which other organism works in conjunction with Mycoplasma hyopneumonia to cause exudative bronchopneumonia and polyarthritis?

A - Bordetella bronchiseptica 
B - Fusobacterium necrophorum 
C - Hemophilus parasuis 
D - Pasteurella multocida 
E - Swine influenza virus
A

d)
Pasteurella multocida infection in conjunction with Mycoplasma Hyopneumoniae causes exudative bronchopneumonia, polyarthrites, and chronic lung lesions.
Mycoplasma hyopneumoniae (also called “enzootic pneumonia”) is a common, smouldering, low-level illness. Stressors (parasites, other infections, even weather) can result in sever pneumonia. Best managed by decreasing stressors with improved ventilation and decreased overcrowding. In endemic herds, ABX for sick individuals (ie: lincomycin, tyrosine, tiamulin, or a tetracycline) helps control illness, most likely by preventing secondary bacterial infection (like Pasteurella). Bacterin vaccines give good protection, decreases signs (coughing). Pre-farriwing vaccination of sows decreases colonization in suckling piglets.

142
Q

A cow is presented on emergency with urea/non-protein nitrogen toxicity.

What is the treatment of choice?

A - Rumenotomy
B - Relieve bloat, drench with 2-8 liters sodium bicarbonate
C - IV Fluids with MgSO4, Na thiosulfate PO
D - Rumenal infusion 2-8 liters vinegar, 3-10 gallons cold water
E - Atropine, Protopam chloride IV q 4-6 hours

A

d)
Treat urea/non-protein nitrogen (NPN) toxicity with a ruminal infusion 2-8 laters 5% acetic acid (vinegar) and 3-10 galons of cold water. The vinegar decreases ruminal pH which slows absorption of un-ionized ammonia. Repeat Q6hours up to 48 hours. Best results if animal is still ambulatory. It is often impossible to treat these cases before they die because of rapid progression to death. If possible, Rx with IV fluids. If necessary, relieve bloat. Urea/NPN toxicity is related to ammoniated feed toxicity which causes so-called “bovine bonkers”. Manage ammoniated feed toxicity by removing the ammoniated feed and treating sever cases with anticonvulsants like diazepam, pentobarbital sodium.

143
Q

For the last ten years, your state has had a mandatory vaccination program against “pedunculated giblet disease” in fur-bearing turtles and the prevalence of this terrible disease has decreased markedly.

How does this decrease in prevalence affect the predictive value positive (PVP) of the best serologic test for pedunculated giblet disease?

A - PVP depends on the number tested, not prevalence
B - PVP is affected by specificity, not prevalence
C - PVP increases as prevalence decreases
D - PVP decreases as prevalence decreases
E - PVP stays the same as prevalence decreases

A

Don’t think too hard on this: As prevalence of a disease goes DOWN, PVP of your serologic test also goes DOWN. That is, as your disease becomes more and more rare, the predictive value of your same old test gets WORSE. That is all you need to know.

144
Q

Under what conditions is a very sensitive test used?

A - Rare disease, Early diagnosis improves prognosis
B - Lethal disease, Highly prevalent disease
C - Treatment does not affect prognosis, Non-infectious diseases
D - Common disease, infectious diseases
E - Zoonoses, untreatable diseases

A

a)
You need a very sensitive test if:
1. Disease is rare (e.g.: BSE);
2. Early Dx improves prognosis (e.g.: HIV in people); or
3. The disease is highly lethal or consequences of missing a case are severe (e.g.: Rabies, Brucellosis, BSE, screw-worm, FMD, EIA).
Remember that a highly sensitive test will have very false negatives. That means if a test is highly sensitive, you can trust a negative test.

145
Q

Acromegaly is associated with what three diseases in cats?

A - Hypoadrenocorticism, Congestive heart failure, Hepatic lipidosis
B - Hyperadrenocorticism, Pleural effusion, Lameness
C - Diabetes mellitus, Cardiomyopathy, Renal disease
D - Prognathism, Pulmonary edema, Pulmonary Hypertension
E - Vertebral spondylosis, 3rd degree heart block, Glomerulonephritis

A

c)
Diabetes mellitus, Cardiomyopathy and Renal disease.
Think first of Diabetes mellitus in older male cats (ave. 10 yr, 90% male) with feline acromegaly.
First presenting sign may be PU/PD, polyohagia of diabetes. Weight gain in an unregulated diabetic cat strongly suggest acromegaly. Think also cardiomyopathy (˜50%; see weakness, dyspnea, systolic murmur, cardiomegaly/CHF, pulmonary oedema). Think also of renal disease (˜50%; see proteinuria, USpG 1.015-1.025, and glycosuria from diabetes).
May see prognathism (long mandible), lameness (esp.cats), marked vertebral spondylosis.

146
Q

At what age do a cow’s permanent 4th incisors begin to erupt in the mandible?

A - 7-8 years
B - 2 to 2.5 years
C - 3 years
D - 3.5 to 4 years
E - 5-6 years
A

d)
3.5 to 4 years. The 4th incisors are actually canine teeth. References for times of eruption vary by about 6 months because the times of tooth eruption vary among individual animals and breed.

147
Q

Two calves aged eight weeks are presented that are both down and extremely weak.

They are depressed and lying in pools of foul-smelling brown diarrhea with a small amount of blood.

One other calf died suddenly the night before with no signs at all.

They are found to be dehydrated, with rectal temperatures of 105.2 and 105.6 F (40.7 40.9 C)..[N=101.5-103.5F, N=37.8-39.7 C], respectively.

Based on the condition at the top of the differential diagnosis list, what is the treatment plan?

A - Immunize well and sick calves and adult cattle with MLV vaccine, antibiotics for sick calves
B - NSAIDS, antibiotics in feed and water
C - IV fluids, NSAIDS, fequent milk feeding, antibiotics if septic
D - Cull sick calves, disinfect feeding areas, prophylactic antibiotics and NSAIDs for well animals
E - Isolate sick calves, immunize, antibiotics, cull those that do not respond to treatment

A

c) IV fluids, NSAIDS, frequent milk feeding and antibiotics. Fever, diarrhea and sudden death in 8-week-old calves is highly suggestive of septicaemia due to salmonellosis. Isolate sick calves. Use of antiobiotics is controversial as they may prolong recovery and shedding and yield a carrier calf. However if an animal is septic it needs parenteral antibiotics. According to Pasquini’s, prognosis is poor with neonatal salmonella and deaths can approach 100% in affected calves. In adults, antibiotics may yield a clinical cure, but Salmonellae can establish a biliary system and intermittently shed into GI system, leading to environmental contamination. Prevention is dependant on which species of Salmonella is causing the problems-host adapted or environmental.

148
Q

What kind of organism causes equine granulocytic ehrlichiosis?

A - Spirochete
B - Anaplasma 
C - Ehrlichia 
D - Protozoa
E - Chlamydia
A

b)
This is a tricky question to help you remember that two diseases formerly classified as ehrlichia have now been re-classified. The causative organism of equine granulocytic ehrlichiosis (EGE) was originally classified as Ehrlichia equine, but is now called Anaplasma phagocytophilum due to DNA sequencing studies. Do not confuses EGE, a necrotizing vasculitis in horse with bovine anaplasmosis, primarily an anemia, with icterus, fever. Another name change occurred with the causative organism of Potomac horse fever (PHF), from Ehrlichia risticii to Neorickettsia risticii. PHF presents as a febrile colitis/diarrhea, with laminitis 3-5 days after diarrhea in horses of all ages: A big rule out is salmonella.

149
Q

A 12-year-old spayed female black Scottish terrier is presented with a three week history of limping on the right forelimb.

The second digit of the paw is enlarged, ulcerated, and bleeding with a loose toenail.

A lytic bone lesion of the 3rd phalanx is visible on radiographs, and cytology of a fine needle aspirate of the mass suggests neoplasia.

If biopsy of the mass confirms malignant subungual melanoma, which one of the following choices contain the best advice to the owner?

A - Long-term prognosis is excellent if the tumor is completely resected.
B - Metastasis is common. Radical excision, plus adjuvant therapies is typically recommended
C - Due to the exquisite sensitivity of melanoma to radiotherapy, it is preferred as a sole treatment
D - DNA-based vaccine encoding canine tyrosinase is effective against oral tumors but not licensed for subungual melanoma
E - Nail bed melanomas in dogs are almost always benign and respond well to cryotherapy.b)

A

b)
Radical excision of a sublingual malignant melanoma mass, with adjuvant therapy such as chemotherapy, radiation, and/or vaccination (off label since it is only approved for oral melanoma), is typically recommended. Melanoma of the digit is an aggressive, infiltrative tutor, with substantial metastatic potential and a poorer prognosis than cutaneous melanoma or other tutors of the digit. Fine needle aspirates of regional lymph nodes and thoracic radiographs should always be performed to assess for metastasis. Even if metastasis. Even if metastasis is not detected, many practitioners still employ adjuvant therapies. The median survival rate for dogs with sublingual melanoma treated with surgical excision is 12 months. The effectiveness of adjuvant therapy protocols in treating sublingual melanomas is not well described. The melanoma vaccina is only approved to treat oral melanoma, and contains DNA for human tyrosinase.

150
Q

A four-year-old male, neutered cat is presented with a history of severe lethargy and inappetance.

The cats mucous membranes are extremely pale and his PCV at presentation is 9%. [Normal=30-45%]

A blood transfusion is indicated.

Which one of the following choices best explains the rationale for blood typing prior to transfusion in cats?

A - Cats have naturally-occurring antibodies against the blood group antigen they lack
B - Blood typing is not necessary. Cats are universal receivers and can receive any blood type
C - Cats that are negative for DEA 1.1 are more likely to have a reaction to DEA 1.1 positive blood
D - If donor is Type A, it is a universal donor and recipient typing is not needed
E - Blood typing will determine if recipient cat is Rh factor negative or Rh positive

A

a)
Cats have naturally-occuriring antibodies against the blood groups antigen they lack. Feline blood groups include A, B and AB (rare).
Type A cats have naturally-occurring anti-B antibodies without any previous exposure to type B antigens. Type B cats have naturally-occurring anti-A antibodies. Type AB do not have antibodies of either group. Type B cats that receive type A blood are particularly susceptible to fatal transfusion reactions as a result of severe acute intravascular hemolysis. Type A cats that receive type B blood have a weaker reaction that commonly results in extravascular hemolysis. Type AB cats should receive type AB or type A blood. Unlike dogs, there are no universal donors in cats.

151
Q

It is noticed on a flow chart that a doctor had entered “ongoing losses WNL” under the comments section. What are “ongoing losses” in fluid therapy?

A - Urine output
B - Fluids given minus extracellular volume estimate
C - Fluid lost during panting
D - Vomiting and diarrhea
E - Percent dehydration multiplied by body weight

A

d)
Ongoing losses are typically vomiting diarrhea, or other losses in a third compartment (e.g., the pleural or abdominal cavity, subcutaneous space). These are sometimes also called “contemporary” losses. Classic examples of diseases with significant ongoing losses are a puppy with parvovirus who has both diarrhea and vomiting or a horse with acute colitis. Sensible losses are easily measured, like the fluids the animal loses by urination. Insensible losses are the fluids an animal loses that are hard to measure, like through breathing and via the skin. In humans and animals that sweat (like horses), sweating is part of insensible losses. Remember that all three types of loss must be accounted for (or at least estimated) when calculating fluids needed to rehydrate a dehydrated animal.

152
Q

Prior to the 1950s, heat pasteurization of milk was done principally to prevent transmission of one organism to people.

Today, that organism is re-emerging in bi-national communities with ties to Mexico who consume unpasteurized milk products.

What is the organism?

A - Mycobacterium bovis
B - Enterotoxigenic Escherichia coli O157:H7
C - Mycobacterium avium subspecies paratuberculosis
D - Mycobacterium tuberculosis
E - Mycobacterium avium

A

a)
A century ago, Mycobacterium bovid (bovine tuberculosis) used to be a common zoonotic health threat, transmitted through milk to children through their emerging teeth at the gums. Today, human cases of M. bovis are emerging more in bi-national communities with ties to Mexico who consume unpasteurized milk products (like fresh quest). Temperature of pasteurization is set by the toughest, most heat-resistant organism, which is Coxiella burnetii, the cause of Q fever. If the temperature is high enough, for long enough to kill Coxiella, it will also kill the other organisms.
Brucella and Listeria can also be transmitted in milk. E coli is typically a contaminant from unsanitary milking conditions.

153
Q

In which equine virus is antigenic drift the most important in the development of novel strains?

A - Adenovirus
B - Herpesvirus-4
C - Herpesvirus-1
D - Influenza virus
E - Papillomavirus
A

d)
Antigenic drift is most important in the epidemiology of equine influenza. Point mutations in the genes for hemagglutinin and neuraminidase cause changes in these surface proteins that can lead to the development of new virus strains not recognized by antibodies to earlier strains. Because influenza is an RNA virus without proofreading ability, mutations can occur more rapidly with flu than mutations seen in DNA viruses like equid herpesviruses and adenoviruses.

154
Q

A seven-year-old dog weighing 24 lbs (10.9 kg) is presented after the owner caught the dog eating warfarin-containing rat poison the day before.

The dog has no overt clinical signs at presentation.

Baseline coagulation parameters are normal [Prothrombin time (PT), Activated partial thromboplastin time (aPTT) and Proteins induced by vitamin K1 absence or antagonism (PIVKA)].

The dog is confined to limit exercise and monitored. At 72 hours PT and PIVKA tests are elevated.

Which one of the following choices is the most appropriate treatment step?

A - Oral vitamin K1
B - Plasma transfusion
C - Blood transfusion
D - Subconjunctival apomorphine
E - Activated charcoal
A

a)
If PT or PIVKA tests elevate at 48 or 72 hors in a case-patient with suspected anticoagulant rodenticide toxicity, start oral vitamin K1 at 1,5-2,5 mg/kg q 12h or 3-5mg/kg once daily. Continue treatment for 14 to 30 days, depending on the specific anticoagulant. Warfarin is a first generation, shorter-acting anticoagulant, brodifacuom is a second-generation long-acting anticoagulant. PT or PIVKA should be checked 48 hours after the last dose of vitamin K1. IF PT or PIVKA is still elevated after the last dose (16 to 32 days post exposure), restart vitamin K1 treatment for 1 week, then recheck PT or PIVKA 48 hours after last dose. Once the PT or PIVKA is normal at 48 hours, you can stop vitamin K1 treatment. Injectable vitamine K1 has no advantages over oral vitamin K1 and can cause anaphylaxis in some animals, even when injected subcutaneously. A blood or plasma transfusion is indicated for animals that present with overt clinical signs (hemorrhage, dyspneic, pale).

155
Q

A cat with a previous diagnosis of diabetes mellitus confirmed by persistent fasting hyperglycemia and persistent glycosuria is presented for a routine check-up.

The owner relates that she ran out of injectable insulin two months ago, but the cat seemed to do fine without it, so she stopped giving insulin shots.

On physical exam, the cat appears healthy and a dipstick test shows a blood glucose level of 125 mg/dL (6.9 mmol/l) [normal 61-132 mg/dL (3.5-7.3 mmol/l)].

What is the most likely explanation?

A - Type I diabetes mellitus
B - Concurrent diabetes insipidus
C - Incorrect initial diagnosis
D - Diabetic remission
E - Insulin resistance
A

d)
Up to 20% of cats may have diabetic remission (sometimes called transient diabetes) characterized by resolution of clinical signs weeks to months after beginning insulin treatment. Clinical diabetes mellitus may or may not recur in the future. Cats with two 2 (non-insulin-dependent) diabetes mellitus may respond to a treatment regimen of weight loss, diet, oral hypoglycemic drugs, and correction of concurrent insulin antagonistic disease; that is the can respond without injectable insulin. Persistant fasting hyperglycaemia (blood glucose greater than 200 mg/dL (11.1mmol/L)) and persistent glycosuria are confirmatory for diabetes mellitus, so “incorrect diagnosis” is a wrong answer in this case.

156
Q

What is the most common clinical sign exhibited by a horse with cystic calculi (bladder stones)?

A - Straining to defecate
B - Colic
C - Hematuria after exercise
D - Uremic breath and urethral ulcers
E - Inappetance
A

c)
Bladder stones in horses are usually single, large speculated stones composed of calcium carbonate. The most common clinical sign exhibited by horses with cystic calculi is hematuria after exercise. hematuria is most evident toward the end of a voided urine stream. Other signs may include strangurie, pollakiuria and urinary incontinence. In adult mal geldings, urethral abstraction may also be associated with cystitis. If there is urethral blockage by smaller uroliths, strangurie, pollakiuria, incontinence, restlessness, colic and stretched posture may be more prominent.

157
Q

A 12-year-old neutered male black labrador retriever is presented with a three week history of limping on the right fore.

The lateral digit is swollen and the nail is deviated ventrally, with ulceration of the nail bed.

A lytic bone lesion of the 2nd phalanx is visible on a radiograph and cytology of the mass suggests neoplasia.

Chest radiographs are clear.

Following amputation of the digit, histopathology indicates that the mass is a squamous cell carcinoma.

Which of the following is the best advice for the owner?

A - 95% chance he will survive 1 year
B - Chemotherapy is indicated
C - Radiotherapy is indicated
D - Guarded prognosis
E - 50% chance he will survive 1 year
A

a)
The prognosis is good. 95% of dogs with digital squamous cell carcinoma (SCC) survive one year after amputation and 75% survive two years after amputation. SCCs have a low metastatic rate; most have not metastasized at the time of diagnosis. Chest radiographs and aspiration of local lymph nodes for cytology are necessary to accurately assess prognosis. In contrast, malignant melanoma of the digit is an aggressive tutor, with substantial metastatic potential and poorer prognosis. Median survival with resection is 12 months, with a 30% local recurrence rate. (That is, only 50% are still alive after 12 months).

158
Q

During the necropsy of an eight-year-old mixed breed dog from the Southern United States, reactive granulomas in the esophagus containing bright red worms, 40 mm to 70 mm long are found.

Which choice is the most likely diagnosis?

A - Ollulanus tricuspis 
B - Haemonchus placei 
C - Spirocerca lupi 
D - Gastrophilus spp.
E - Physaloptera spp.
A

c)
A disease of dogs in the Southern US and tropical climates, Spirocerca Tupi (esophageal worms) make reactive granuloma of variable size in the esophageal, gastric or aortic walls. Spirocercosis may also lead to aneurysm in the thoracic aorta or an ossifying spondylitis of the posterior thoracic vertebrae. Typically asymptomatic, but large granulomas can cause esophageal obstruction. Large granulomas may become neoplastic (osteosarcoma, fibrosarcoma). Some dogs develop spondylitis or enlargement of the extremities characteristic of hypertrophic osteopathy.
All of the other choices are gastric parasites. Ollulanus tricuspid in an uncommon gastric parasite of cats. Phylasopter spp is a stomach worm tat may cause vomiting, anorexia, dark faces in dogs and cats. Haemonchus spp, Ostertagia and Trichostongylus are found in the abomasum of ruminants. Gasterophilus are the larvae of horse bot flies, found in the stomach of horses.

159
Q

A mare is presented with a history of intermittent lameness in the right fore involving a shortened stride and occasional stumbling.

She relieves pain by pointing the affected foot with the heel off the ground. A flexion test of the distal right forelimb exacerbates the lameness temporarily.

Which one of the following choices is the most likely diagnosis?

A - Laminitis
B - Caudal heel pain
C - Villonodular synovitis
D - Pedal osteitis
E - Sidebone
A

b)
Caudal heel pain (also called navicular disease) is an intermittent-onset lameness with a poorly understood pathogenesis. The horse may point affected foot with the heel off the ground to relieve the pressure of the deep digital flexor tendon on the painful navicular area. If both fores are affected, a horse may alternate pointing. Typically presents as an intermittent lameness early in the course of disease, with a shortened stride. The horse may tend to stumble. A flexion test of the distal affected forelimb can usually exacerbate the lameness temporarily.

160
Q

A positive serology for Sarcocystis neurona antibodies in the serum of a horse indicates which one of the following?

A - Favorable response to treatment for equine protozoal myeloencephalitis
B - Definitive diagnosis of equine protozoal myeloencephalitis
C - Poor response to treatment for equine protozoal myeloencephalitis
D - Exposure to Sarcocystis neurona
E - The horse has been in Africa

A

d)
There is no definitive ante mortem test for equine protozoal myeloencephalitis (EPM). Serologic tests including evaluation for surface antigens (snSAGs) and the indirect fluorescent antibody test (IFAT) show that a horse has been exposed to the organism. Positive serum results acne are difficult to interpret (can indicate exposure) but negative serum titers are often considered diagnostic to rule out EPM except in very acute cases of disease. A ration of serum to CSF antibody showing intrathecal antibody production against S.neurona combined with clinical suspicion is highly suggestive of a diagnosis of EPM. Definitive diagnosis requires post-mortem examination.

161
Q

A group of feeder pigs is presented for strange behavior.
The pig farmer complains they are hyperexcitable yet are not squealing.
A few are lethargic, wandering aimlessly, and seem to be blind.

Bloodwork shows a marked increase in sodium concentration.

Upon inspection of the pen, it turns out that the waterer had been inadvertently turned off.
Which one of the following treatments is indicated?

A - Frequent small amounts of water
B - Ad lib H2O access
C - Mannitol
D - Furosemide
E - Hypertonic saline solution IV, rapid bolus
A

a)
It is dangerous to rapidly correct for a hyperosmolar or hypoosmolar state (salt toxicity)– could lead to brain swelling. It is best to gradually rehydrate the animals with small amounts of water given frequently. Most pigs showing clinical signs usually die regardless of treatment for salt poisoning and you are only trying to decrease mortality across the herd. Some references advocate use of slow infusion of slightly hypertonic IV fluids to decrease cerebral edema. This requires intensive care with frequent measurements of blood sodium concentrations. This may apply to pets, but is not realistic in a farm situation with multiple pigs.

162
Q

A six-year-old female spayed cocker spaniel is presented with a two-day history of lethargy. Upon physical exam mucosal petechiae and ecchymoses and an ocular hemorrhage in the right eye are noted.

A coagulation profile shows the following:
Thrombocytes= 12,720 [N=200,000-900,000].
Buccal mucosal bleeding time (BMBT), increased
Activated partial thromboplastin time (aPTT), normal
Prothrombin time (PT), normal
Thrombin time (TT), normal

Which one of the following diseases is the most likely diagnosis?

A - Hepatic insufficiency
B - Von Willebrand's disease
C - Anticoagulant rodenticide toxicity
D - Disseminated intravascular coagulation (DIC)
E - Immune-mediated thrombocytopenia
A

e)
The history, plus a lab pattern of low platelets, increased bleeding time and normal aPTT, PT, TT tests suggests immune-mediated thrombocytopenia the most common cause of spontaneous bleeding in dogs. It can be a primary disorder (idiopathic) or secondary to infections, neoplasia or certain drug therapies. Twice as common in females; cockers, poodles and old English sheepdogs are predicted, but can occur in any breed.
Dogs with disseminated intravascular coagulation (DIC) often present with thrombocytopenia but will also typically have a prolonged PT and aPTT.

163
Q

Oxalate is the toxic agent found in which one of the following plants?

A - Hypericum perforatum (klamathweed)
B - Delphinium spp. (larkspur)
C - Sarcobatus vermiculatus (greasewood)
D - Hordeum spp. (foxtail)
E - Solanum spp. (nightshade)
A

c)
Oxalates are found in Sarcobatus vermiculatus (greasewood).
Greasewood toxicity is seen primarily in sheep, sometimes in cattle, usually when large amounts are consumed over a short period of time. Oxalates occur in plants as salts of calcium, sodium, and potassium. Calcium oxalate is insoluble and is lost through the GI tract. Sodium and potassium oxalates are soluble and either 1) bind to calcium in the rumen and lost through the GI tract, or 2) are absorbed and react with calcium in body fluids. Death is attributed to hypocalcémie and/or kidney failure caused by calcium oxalate crystals in the renal tubules. Signs of oxalate toxicity include: dullness, lowering of the head, loss of appetite, operation from the herd; followed by excessive salivation with frothing, progressive incoordination; and finally, coma, irregular breathing, and death. Oxalate containing plants include: Sarcobatus (greasewood), Oxalis (sorrel) Rumex (dock), Halogeton, Amaranthus (pigweed), ans Chenopodium (lambs quarter). Oxalates are also produced by molds (Aspergillus niger) in contaminated feeds.
Delphinium spp (larkspur) contain alkaloids. Hypericum perforated (klamathweed) contain hypericin.
Hordeum spp (foxtail) cause physical injury.
Solanum spp( nightshade) contain alkaloids.

164
Q

Several piglets in a group weaned ten days ago in the nursery facility of a large commercial swine operation were found dead.

On evaluation, some weaners have swelling around the eyes and forehead.

Some are in lateral recumbency and dyspneic.

Necropsy of the dead piglets reveals subcutaneous and submucosal edema.

What is the most likely causative organism?

A - Brachyspira hyodysenteriae 
B - Escherichia coli 
C - Lawsonia intracellularis 
D - Clostridium septicum 
E - Streptococcus suis
A

b)
E. coli causes the characteristic lesions of edema disease on recently weaned piglets. Marked swelling of the periocular region, forehead and submandibular area follow infection. Piglets may die peracutely. Usually only a few piglets in a group are affected, but affected piglets perish rapidly (within 12 hours). Hemolytic E.coli that produce F18 pili and Shiga toxin 2e are implicated in deem disease. To make a definitive diagnosis, E.coli must first be isolated and then characterized as an edema disease strains (that is, producing F18 pili and Shiga toxin 2e). The course is so rapid that treatment is ineffective. Antibiotics may be administered to unaffected pigs in the group.
Brachyspira hyodysenteriae causes Swine dysentery also called bloody scours. Lawsonia intracellular causes Porcine proliferative enteritis (diarrhea, often with fibronecrotic casts). Clostridium septum is the agent of Malignant edema in many animals. Infection occurs through contaminated wounds and turns affected muscle dark brown of black. Streptococcus suis causes septicaemia and meningitis in wearers and growing pigs.

165
Q

Syndactyly is most commonly found in which of the following cattle breeds?

A - Brown Swiss
B - Angus
C - Hereford
D - Holstein-Friesian
E - Simmental
A

d)
Holstein-Friesian cattle inherit syndactyle, or “mule foot”, a simple autosomal recessive trait, more often than other breeds. Syndactyly is the partial or complete fusion of the digits of one or more feet. Forefeet are affected more often. Affected animals walk slowly, usually with a high-stepping gait. They may also be more prone to hyperthermia than normal cows.

166
Q

Which one of the following organisms transmits the causative agent of blackhead disease in turkeys?

A - Heterakis gallinarum 
B - Ascardia galli 
C - Capillaria spp
D - Eimeria spp
E - Raillietina spp
A

a)
The cecal nematode Heterakis gallinarum transmits Histomonas meleagridis (a protozoan), the causative agent of blackhead disease. Clinical signs of histomoniasis include listlessness, droopy wings, unkempt feathers, yellow droppings.
Clinical signs of Ascardia galli infection include diarrhea, anemia, and decreased egg production. Birds infected with Capillaria or Eimeria have hemorrhagic enteritis. Clinical signs of Raillietina (tapeworm) infection range from none to enteritis.

167
Q

Which one of the following choices includes the cardinal sign of trigeminal neuritis?

A - Dysphagia, dysphonia and stridor
B - Circling and head tilt toward side of lesion, no other signs
C - Masseter muscle pain associated with chewing
D - Paralyzed eyelid, ear or lip on one or both sides of the face
E - Inability to close the mouth

A

e)
Idiopahtic trigeminal neuritis, due to inflammation of cranial nerve 5 (CN 5), is characterized by acute onset of flaccid jaw paralysis. Affected animals cannot close their mouth and have difficulty eating and drinking. Seen occasionally in dogs, rare in cats. Cause is unknown. Idiopathic facial nerve paralysis, affecting cranial nerve 7 (CN 7), results in the inability to move the eyelid, lip, or eat and dryness of the eyes and mouth. Masticatory myositis is characterized by pain on opening the mouth and swelling of the muscles of mastication (acute) or atrophy of the temporals and master muscles with the inability to open the mouth due to fibrosis (chronic). Dysphagia, dysphonia, and stridor are most often associated with dysfunction of the vagus never, cranial nerve 10 (CN 10). Cricling and head tilt toward the side of the lesion with no other signs is a common presentation of vestibulocochlear nerve, cranial nerve 8 (CN8) lesions. Concurrent CN 7 paralysis and Horner’s syndrome (ptosis, moisis, exophthalmos) may be present with middle and inner ear infections.

168
Q

An outbreak of diarrheal disease of piglets has occurred which affected the healthiest animals in the herd, one to two weeks after weaning.

Some affected piglets had no signs except peracute death.

Other affected piglets exhibit diarrhea, ataxia, paralysis, and recumbency.

What condition is at the top of the differential diagnosis list?

A - Edema disease
B - Epidemic transmissible gastroenteritis (TGE)
C - Hemagglutinating encephalomyelitis virus (HEV)
D - Clostridium perfringens type C enteritis
E - Porcine proliferative enteritis

A

a)
Edema disease is caused by shiva toxin-producing E.coli (STEC). Look for sever acute illness ranging from percute death with no signs to CNS involvement with ataxia, paralysis, and recumbency in healthiest pigs 1 to 2 weeks after weaning. Hemagglutinating encephalomyelitis virus (HEV), is almost exclusive to piglets less than 4 weeks old. Two clinical presentations: vomiting and wasting disease (VWD) and encephalitic. Clostridium perfringens type C enteritis, also called enterotoxemia in other animals, is characterized by a hemorrhagic diarrhea in 1 to 3 day-old piglets. Porcine proliferative enteritis is principally a diarrheal disease of growing finishing (40 to 80 lb) pigs and young breeding pigs. Epidemic transmissible gastroenteritis (TGE) in non-immune pig herds characterized by high morbidity and high mortality in piglets less than 1 week old.

169
Q

What condition is associated with rectal stricture in pigs?

A - Coccidiosis
B - Salmonella typhimurium
C - Intestinal spirochetosis
D - Intussusception
E - Rotavirus
A

b)
Salmonella typhimurium is associated with rectal strictures in growing pigs. Caused by an ulcerative proctitis that damages rectal tissue. Can see large number of cases. Can see sporadic rectal strictures as a sequelae to rectal prolapse. Intestinal spirochétoses is a post-weaning diarrhea seen in the absence of Brachyspira (serpulinaj) hyzdysenteriae (swine dysentery), but similar in presentation to it. This syndrome is being recognized more frequently worldwide.

170
Q

Which of the following is the correct location for placement of a proximal paravertebral block used to perform a standing laparotomy in a cow?

A - In the spaces between L1, L2, L3, and L4
B - Above and below the transverse processes of L1, L2 and L4
C - Caudal to the transverse processes of T13, L1, and L2
D - Midway between spine and the ends of the transverse processes of L1, L2, L3
E - At the ends of the transverse processes of L1, L2, and L4

A

c)
A proximal paravertebral block is performed via placement of local anesthetic just off the midline and caudal to the transverse processes of T13, L1, and L2 vertebrae. With a distal paravertebral block, local is placed above and below the ends of the transverse processes of L1, L2, and L4 vertebrae. The spinal nerves T13, L1 and L2 are targeted by a paravertebrl block to completely desensitize the Planck of a cow. Note the difference- proximal at T13, L1 and L2 distal at L1, L2 and L4, but the same nerves are affected because they gradually course caudally after exiting their spinal Forman. Proper placement results in warming of the skin from vasodilation, anesthesia of the skin and body wall, and a curvature of the spine in some cows. The latter is caused by relaxation of the epaxial musculature on the affected side; the spine curves towards the opposite side.

171
Q

A high-producing dairy cow that freshened 3 weeks ago is off-feed. On physical exam her heart and respiratory rates are withing normal limits. T=101.2 F (38.4 C) [N=100.4–102.8 F; 38.0–39.3 C]

Rumen motility is decreased and a urine test for ketone bodies is positive. There is no evidence of mastitis and the uterus is clear of infection.

On the left side, a high-pitched musical “ping” is audible via stethoscope during percussion over the ribs on a line between the elbow and tuber coxae (hip).

What acid-base abnormality is most likely in this cow?
A - Metabolic alkalosis
B - Depends on severity of displacement
C - Respiratory acidosis
D - Respiratory alkalosis
E - Metabolic acidosis
A

a)
Think of hypochloremic metabolic alkalosis due to hydrochloric acid
(HCl) sequestration in the abomasum of a cow with a displaced abomasum. Abomasal hypo motility, ongoing HCl secretion into the abomasum, and partial abomasa outflow obstruction all contribute. Metabolic alkalosis can also be due to HCl loss in mono gastric animals who vomit. In a similar way metabolic acidosis can be due to HCO3 loss in saliva if animal cannot swallow, or from diarrhea.

172
Q

A nine-year-old German shepherd is presented with unchecked bleeding from a cut on the gums above the right canine tooth. The owner relates that the dog has lost weight and had an episode of collapse three days ago, but he recovered.

On physical exam, the gums are pale with petechiae and ecchymotic hemorrhages. There is tachycardia and a palpable cranial abdominal mass.

A coagulation profile shows the following:

Thrombocytes= 82,533 per microliter..[N=200,000-900,000]
Buccal mucosal bleeding time (BMBT), increased
Activated partial thromboplastin time (aPTT), increased
Prothrombin time (PT), increased
Thrombin time (TT), increased
Fibrin degradation products (FDPs), increased

What disorder of coagulation best fits this pattern?

A - Idiopathic thrombocytopenia
B - Von Willebrand's disease
C - Disseminated intravascular coagulation
D - Anticoagulant rodenticide toxicity
E - Hepatic insufficiency
A

c)
A lab pattern of low platelets, increased bleeding time and across the board increases in aPTT, Pt, TT and FDP tests suggests disseminated intravascular coagulation (DIC). DIC is not a disease in tis own right- it is a complex hemostatic defect characterized by enhanced coagulation and fibrinolysis, secondary to other diseases. Fibrinolysis and depletion of clotting facts leads to hemorrhage. Many, many diseases, all of them bad, can precipitate DIC. This case presentation (pale, older German Shepherd with Hx of collapse, bleeding and an abdominal mass) suggests hemangiosarcoma. Remember your “ H diseases” associated with DIC: Heartworm, Heart failure, Hemolytic anemia, Hemangiosarcoma, Hemorrhagic gastroenteritis, Hepatic disease, especially hepatic lipidosis in cats. Gastric dilatation-volvulus (GDV), mammary gland carcinoma and pancreatitis can also lead to DIC.

173
Q

A practice is using an FeLV test with a sensitivity of 90% and a specificity of 95%.

Assuming the prevalence of feline leukemia in the area is 5%, what is the predictive value negative (PVN) of the test?

a) 90%
b) 60%
c) 88&
d) 99%
e) 98%

A

d)
PVN is 99%. The trick with this kind of question is to pick an imaginary number of animal that you test, like 1000, and fill out your 2x2 table from there.

174
Q

A four-year-old Quarter horse gelding is presented with a three-day history of poor appetite, and stiffness in the hind limbs.

Equine infectious anemia (EIA) is on the differential diagnostic list.

What other diagnosis should be considered for a horse with edema, icterus and fever?

A - Equine viral arteritis (EVA)
B - Purpura hemorrhagica
C - Equine influenza
D - Equine viral rhinopneumonitis (equine herpes-1)
E - African horse sickness
A

a)
A horse with edemia, icterus and fever should make you think first of equine infectious anemia (EIA) and equine viral arteritis (EVA). Conjunctivitis and rhinitis+ ventral edema says EVA in this case. Can see limb edema with equine granulocytic ehrlichiosi (caused by anaplasma phagocytophilum), but that choice is not on the list. Respiratory presentation with fever, cough and eyelid swelling in a horse shipped from spain or africa should set off reportable alam belle in your head for african horse sickness.